PDA

Archiv verlassen und diese Seite im Standarddesign anzeigen : Gravitative Sphärenproblematik


Eyk van Bommel
14.08.13, 15:00
Ich verstehe es immer noch nicht so richtig. Da jedoch das Problem beim „letzten Mal“ in höhere Sphären „abrutschte“ – versuche ich mein Problem noch einmal darzustellen, da ich immer wieder darüber stolpere – zuletzt hier (http://scienceblogs.de/hier-wohnen-drachen/2013/08/13/die-vakuumenergie-und-der-casimir-effekt/).
Wie kann eine 4D „Raumzeit-Sphäre“ sich in einem >4-Dimensionalenraum zusammenziehen, wenn die Kraft nur in 4D wirken kann?
Wenn man beim Ballon-Modell bleibt, dann bewirken die „2D wirkenden“ intermolekularen Kräfte zwar das der Ballon (wenn er Luft verliert) sich wieder zusammenzieht, aber dies geschieht doch nur, weil die „Nettokraft“ 3-dimensional wirkt.
Wäre das Universum von „Flatland“ zu einer 3D Sphäre/ oder xy- Donut gekrümmt, dann würde die Gravitation max. dazu führen können, dass sich alle Masse an einem einzelnen Punkt auf der 3D Sphäre sammelt. Das wäre dann eher so wie hier (http://www.youtube.com/watch?v=kag4tU6B3yc). Der „Big Crunch“ würde irgendwo beginnen und das Universum würde „in sich zusammenstürzen“, wobei Ur- und „End“-knall nicht am selben Ort stattfinden. Merke gerade, dass Video ist besser (http://www.youtube.com/watch?v=TdMIsCF_7p0)– beim andern war der Fokus wohl verstellt. :rolleyes:
Ob es aber zum „Big Crunch“ bzw. „Endknall“ kommt hängt dann NICHT von der Energie/Masse im Universum ab – sondern von der Symmetrie der (Massen-)Punkte auf dem Ballon. In einem (am Ende) symmetrischen Universum würde sich die Gravitationskraft aufheben und es kommt nicht zum „Big Crunch“ bzw. „Endknall“.

Gruß
EvB

Bauhof
14.08.13, 15:46
Wäre das Universum von „Flatland“ zu einer 3D Sphäre/ oder xy- Donut gekrümmt, dann würde die Gravitation max. dazu führen können, dass sich alle Masse an einem einzelnen Punkt auf der 3D Sphäre sammelt.

Hallo Eyk van Bommel,

ja, die Gravitation kann nicht für die mögliche Zusammenziehung der 3-D-Sphäre verantwortlich sein, denn alle Massen würden in einem Punkt auf der 3-D-Sphäre kollabieren. Und die 3-D-Sphäre bliebe dabei gleich groß. Warum? Weil wir annehmen, dass die Gravitation nur im dreidimensionalen Raum wirkt und nicht in eine vierte Richtung, die senkrecht auf den 3-D-Raum steht

Meine Lösung: Nachdem die Gravitation nicht für das Größerwerden der 3-D-Sphäre ursächlich sein kann, dann kann ist sie auch nicht im Umkehrschluss für eine Abbremsung der Ballonvergrößerung verantwortlich sein. Es muss etwas anderes sein, was die Welt auseinandertreibt und möglicherweise wieder zusammenzieht.

M.f.G. Eugen Bauhof

Ich
14.08.13, 23:03
Wie kann eine 4D „Raumzeit-Sphäre“ sich in einem >4-Dimensionalenraum zusammenziehen, wenn die Kraft nur in 4D wirken kann?
Raumzeit zieht sich nicht zusammen, und schon gar nicht in einer weiteren Dimension. Wenn man von solchen Sphären spricht, sind diese dreidimensional.
Wenn man beim Ballon-Modell bleibt, dann bewirken die „2D wirkenden“ intermolekularen Kräfte zwar das der Ballon (wenn er Luft verliert) sich wieder zusammenzieht, aber dies geschieht doch nur, weil die „Nettokraft“ 3-dimensional wirkt.

Ja, so ist das im Modell. Andere reden gerne von einem Gummituch, da erfolgt die Kontraktion in 2D.
Wäre das Universum von „Flatland“ zu einer 3D Sphäre/ oder xy- Donut gekrümmt, dann würde die Gravitation max. dazu führen können, dass sich alle Masse an einem einzelnen Punkt auf der 3D Sphäre sammelt.
Sagt wer? Die Newtonsche Gravitation, nehme ich an. Richtig. Die ist aber auch nicht vereinbar mit anderen Topologien wie eben der Sphäre. Das sind zwei unterschiedliche Welten. Newton lebt im unveränderlichen flachen dreidimensionalen Raum. Da kann nichts gescheites herauskommen, wenn man damit die Expansion von Sphären zu beschreiben versucht.
In einem (am Ende) symmetrischen Universum würde sich die Gravitationskraft aufheben und es kommt nicht zum „Big Crunch“ bzw. „Endknall“.
Es gibt keine "Gravitationskraft". Das ist nur eine Scheinkraft. Das Konzept ist geeignet für die Beschreibung lokaler Vorgänge, es ist einfach ungeeignet für die globale Beschreibung eines sphärischen Universums.

Du musst für solche Dinge einen Schritt zurückgehen und diese angebliche Gravitationskraft aus der Beschreibung rausnehmen, die stört am Ende nur.

Was du lokal, also in einem kleinen Raumgebiet hast, ist folgendes: Stell dir eine Wolke Kaffebohnen in Schwerelosigkeit vor. (Ich weiß nicht, warum es Kaffeebohnen sein müssen.) Die schweben da einfach so, kräftefrei, zueinander ruhend.
Jetzt denken wir uns Gravitation dazu. Die Masse des Materials in der Wolke selbst wird dazu führen, dass sie sich zusammenzieht. Ihr Volumen wird mit der Zeit kleiner.
Das ist eine grundlegende Beschreibung, sie gilt sowohl in der Newtonschen Gravitation als auch in der ART. Das ist die Wirkung von Gravitation.
In einem homogenen Universum, das überall gleich aussieht, wird auch überall das Gleiche passieren. Überall kontrahiert die Wolke, überall werden die Abstände mit der Zeit kleiner - proportional zum Abstand selbst. So funktioniert Gravitation. Immer, auch in sphärischen Universen.
Sie funktioniert nicht über Kräfte, die sich gegenseitig aufheben könnten. Ganz egal, was der Rest des Universums macht: wenn innerhalb einer solchen - sonst kräftefreien - Wolke Gravitation wirkt, dann zieht sie sich zusammen, dann wird ihr Volumen kleiner. Das ist so, fertig aus, nichts zu diskutieren, keine Ausnahmen. Alles, was dem widerspricht, beschreibt Gravitation nicht richtig.
Das gilt auch, wenn die Wolke ein ganzes sphärisches Universum ausfüllt. Die Wirkung der Gravitation ist es, die Abstände der Teilchen zueinander zu verkleinern. Und damit die ganze Sphäre.

Eyk van Bommel
15.08.13, 09:06
Hallo Bauhof,
Da sind wir mal ähnlicher Meinung. :eek: :)
Es muss etwas anderes sein, was die Welt auseinandertreibt und möglicherweise wieder zusammenzieht.
Wenn es etwas anderes ist, dann kann diese (konstante) Kraft fast beliebig klein/groß sein und das Universum würde sich trotzdem immer schneller ausbreiten. Die Ausbreitung würde sich nur nicht weitre beschleunigen.
Es muss etwas anderes sein, was die Welt auseinandertreibt und möglicherweise wieder zusammenzieht.
Oder anhalten lässt.

Hallo ICH,
ich dachte/denke nicht, dass ich dem Newtonschen Weltbild (allzu sehr) verhaftet bin. Für mich wäre/ist die Donut bzw. Sphärenform einfach die natürliche Form der „Raumzeitmaterie“Verteilung. Das "2p-1/+1 Orbitalmodell" (http://commons.wikimedia.org/wiki/File%3AHydrogen_eigenstate_n2_l1_m-1.png) des „Gravitationsfelds“ in einem n-Dimensionalen Universum. Die „Aufenthaltswahrscheinlichkeit der Materie “in einem n-Dimensionalen Universum in seinem „Feld“.

Und so wie das e- im Orbitalmodell keine ("von sich aus" messbare Kraft) erfährt – so bewegt sich alle Masse und Energie innerhalb diese der vorgegeben Bahnen was wir als 3D+t Raum wahrnehmen.
Es gibt keine "Gravitationskraft"
Nein ist hautsächlich die „zeidilatative“ Wirkung mit einem „hauch“ von „Raum“krümmung.
Du musst für solche Dinge einen Schritt zurückgehen und diese angebliche Gravitationskraft aus der Beschreibung rausnehmen, die stört am Ende nur.
Ich denke das habe ich.
Trotzdem sehe ich doch die Kaffebohnen/Masse als „e-„ und die Raumzeit als Feld, welches das Orbital (die Form des Universums) bedingt. Und auch wenn sich alle e- im Atom als Cooper-Paar zusammenballen, so bleibt das Orbital erhalten.
Das ist eine sehr „Quantenweltische“ Beschreibung – aber besser kann ich es nicht.
Überall kontrahiert die Wolke, überall werden die Abstände mit der Zeit kleiner - proportional zum Abstand selbst. So funktioniert Gravitation. Immer, auch in sphärischen Universen.
Für mich kollabiert nur nicht gleich das Orbital nur weil es lokale Anhäufungen gibt.

Gruß
EvB

Timm
15.08.13, 13:00
Hallo Eugen,

Meine Lösung: Nachdem die Gravitation nicht für das Größerwerden der 3-D-Sphäre ursächlich sein kann, dann kann ist sie auch nicht im Umkehrschluss für eine Abbremsung der Ballonvergrößerung verantwortlich sein. Es muss etwas anderes sein, was die Welt auseinandertreibt und möglicherweise wieder zusammenzieht.

Das kann ich nicht nachvollziehen. Ein materiehaltiges (anziehende Gravitation) Universum kann je nach dem Anteil abstoßend wirkender 'Gravitation' an der gesamten Energiedichte dennoch unendlich expandieren oder kollabieren.

Gruß, Timm

Timm
15.08.13, 13:41
(Ich weiß nicht, warum es Kaffeebohnen sein müssen.)
Da habe ich John Baez (http://math.ucr.edu/home/baez/gr/ricci.weyl.html) im Verdacht. Aber vielleicht schwimmt der auch im Strom.

Bauhof
15.08.13, 16:45
Das kann ich nicht nachvollziehen. Ein materiehaltiges (anziehende Gravitation) Universum kann je nach dem Anteil abstoßend wirkender 'Gravitation' an der gesamten Energiedichte dennoch unendlich expandieren oder kollabieren.

Hallo Timm,

meinst du mit dem " Anteil abstoßend wirkender 'Gravitation' " an der gesamten Energiedichte die sogenannte "Dunkle Energie" ?

M.f.G. Eugen Bauhof

Timm
15.08.13, 20:10
Hallo Timm,

meinst du mit dem " Anteil abstoßend wirkender 'Gravitation' " an der gesamten Energiedichte die sogenannte "Dunkle Energie" ?

M.f.G. Eugen Bauhof
Ja, bzw. die Kosmologische Konstante. Alles was für negativen Druck verantwortlich ist.

Gruss, Timm

Eyk van Bommel
19.08.13, 08:22
Manchmal rede ich ein bisschen viel (Donut-Orbitalmodell des Universums) aber,

wenn ich mir die schwebenden Kaffeebohnen (wie von dir beschrieben) vorstelle, so sehe ich nachwievor wie sich das G-Feld/die Raumzeit mit c von den Kaffebohnen entfernt und das Universum sogar wachsen lässt.

Selbst wenn die Masse des Universum dazuführt, dass mathematisch sich die Materie „wieder“ zu einem gemeinsamen Punkt zusammen ziehen wird, so sehe ich die Größe des Universums nicht davon beeinflusst.

Gruß
EvB

Bauhof
19.08.13, 09:46
Was du lokal, also in einem kleinen Raumgebiet hast, ist folgendes: Stell dir eine Wolke Kaffebohnen in Schwerelosigkeit vor. (Ich weiß nicht, warum es Kaffeebohnen sein müssen.) Die schweben da einfach so, kräftefrei, zueinander ruhend.
Jetzt denken wir uns Gravitation dazu. Die Masse des Materials in der Wolke selbst wird dazu führen, dass sie sich zusammenzieht. Ihr Volumen wird mit der Zeit kleiner.
Das ist eine grundlegende Beschreibung, sie gilt sowohl in der Newtonschen Gravitation als auch in der ART. Das ist die Wirkung von Gravitation.

Hallo ICH,

in einem der möglichen Universum-Modelle ist der 3-D-Raum eine in sich zurückgeschlossene 'Sphäre'. Genauer: die in sich selbst zurückgeschlossene Sphäre ist der dreidimensionale Begrenzungsraum eines vierdimensionalen Gebildes – rein mathematisch gesehen.

Nun seien deine "Kaffeebohnen" in etwa gleichmäßig in dieser Sphäre verteilt. Mit der Newtonschen Theorie wird man es nicht beschreiben können, dass beim "großen Kollaps" sich auch die Sphäre zusammenzieht. Nur die "Kaffeebohnen" werden sich an einem oder an mehreren Orten im 3-D-Raum versammeln und kollabieren.

Ich kann mir vorstellen, dass mit Hilfe der ART beim großen Kollaps auch die Zusammenziehung der Sphäre beschrieben werden kann, weil die Massen auf die Raumzeit rückwirken. Aber wie? Kann man das intuitiv erfassen, ohne die Mathematik zu bemühen?

M.f.G. Eugen Bauhof

Timm
19.08.13, 20:06
wenn ich mir die schwebenden Kaffeebohnen (wie von dir beschrieben) vorstelle, so sehe ich nachwievor wie sich das G-Feld/die Raumzeit mit c von den Kaffebohnen entfernt und das Universum sogar wachsen lässt.

Selbst wenn die Masse des Universum dazuführt, dass mathematisch sich die Materie „wieder“ zu einem gemeinsamen Punkt zusammen ziehen wird, so sehe ich die Größe des Universums nicht davon beeinflusst.



Diese Fragen klären sich, wenn man bei dem ja nicht falschen Bild des expandierenden bzw. kontrahierendem Raums bleibt. Dann nehmen die Entfernungen zwischen den Objekten zu bzw. ab. Dann führt die Materie kein Eigenleben unabhängig von der Größe des Raums, sondern ist 'mitbewegt'.

Eyk van Bommel
22.08.13, 07:12
Das Problem beim Gummituchmodell ist, dass die Dehnung im unendlichen null wird. Egal wie stark die Masse lokal das Gummituch „eindrückt“ der Rand würde keinen Zug verspüren. Zudem entfernt sich der Rand mit c – die „Information des Dehnens“ erreicht ihn nie.

Auch wenn das Gummituch eine Sphäre bildet - so sehe ich nur ein Gummituch das „immer dünner“ wird und nicht einen Ball der kleiner wird.

Gruß
EvB

Timm
22.08.13, 10:07
Du scheinst da ein Problem zu sehen, wo keins ist, Eyk.

Lokale Inhomogenitäten ändern an der globalen Massendichte und damit an der dynamischen Entwicklung des Universums nichts.

Gruß,Timm

Eyk van Bommel
22.08.13, 13:12
Hallo Timm,
Mir geht es auch nicht um „lokale Inhomogenitäten“. Sondern darum was passiert wenn die Materie sich „wieder“ an einem Punkt trifft. Ich verstehe nicht wie die (vorlaufende) Ausbreitung des(/der) Gravitationsfelds(änderung) mit c - durch den Kollaps der Materie aufgehalten werden soll.

Kurz: Ich habe dasselbe Problem wie Bauhof.
(Zitat)
Ich kann mir vorstellen, dass mit Hilfe der ART beim großen Kollaps auch die Zusammenziehung der Sphäre beschrieben werden kann, weil die Massen auf die Raumzeit rückwirken. Aber wie?
Kann man das intuitiv erfassen?
Ich verstehe es nicht beim Gummituch und somit auch nicht beim Ballon-Modell. Das Gummituch ist ja beliebig dehnbar. Daher kann die Masse sich zwar an einem Ort „treffen“ die Größe des Gummituchs wird dadurch nicht geändert.

Gruß
EvB

Bauhof
22.08.13, 13:56
Ich verstehe es nicht beim Gummituch und somit auch nicht beim Ballon-Modell. Das Gummituch ist ja beliebig dehnbar. Daher kann die Masse sich zwar an einem Ort „treffen“ die Größe des Gummituchs wird dadurch nicht geändert.

Hallo Eyk van Bommel,

wenn die Massen auf das "Gummituch" rückwirken könnten, dann könnte sich beim "großen Zusammensturz" auch das Gummituch verändern. Das könnte der Fall sein, wenn man die ART zugrundelegt, wie bereits von mir angedeutet.

Bei der Zugrundelegung der Newtonschen Gravitationstheorie gibt es m.E. keine Rückwirkung auf das Gummituch. Dann treffen sich die Masse beim großen Zusammensturz an einem oder an mehreren Orten, ohne dass das (sphärische) Gummituch kleiner wird. So wie du es annimmst. Ob dabei das Gummituch dehnbar ist oder nicht, spielt beim Newtonschen Fall keine Rolle.

Nun bleibt die Frage, ist der große Zusammensturz nach Newton beschreibbar oder muss man ihn nach Einstein beschreiben. Und vor allem bleibt die Frage, warum man ihn nicht nach Newton beschreiben kann.

M.f.G. Eugen Bauhof

Timm
22.08.13, 18:15
Hallo Eugen,

Nun bleibt die Frage, ist der große Zusammensturz nach Newton beschreibbar oder muss man ihn nach Einstein beschreiben. Und vor allem bleibt die Frage, warum man ihn nicht nach Newton beschreiben kann.

Ich denke, daß in Newton's absolutem Raum eine endliche homogene Materieverteilung auf ein Zentrum hin kollabiert, eine unendliche aber nicht. Denn wo wäre da das Massenzentrum. Die Materie verhält sich unabhängig vom Raum.

Bei der ART sind Raum und Energiedichte miteinander verwoben, sodaß der Anteil der Materiedichte auf die Dynamik des Raums wirkt. Und das unabhängig davon, ob das Universum endlich oder unendlich ist. Die Gültigkeit des kosmologischen Prinzips muß man wohl hinnehmen. Man kann das guten Gewissens tun angesichts des Erfolgs des Lambda-CDM Modells. Deshalb erfolgt nach der ART der "große Zusammensturz" überall. Worauf ja auch 'Ich' schon hingewiesen hat. Nach Newton gibt's bei einem Zusammensturz ein Zentrum. Der Raum merkt davon nichts.

Gruß, Timm

Bauhof
23.08.13, 13:03
Bei der ART sind Raum und Energiedichte miteinander verwoben, sodaß der Anteil der Materiedichte auf die Dynamik des Raums wirkt. Und das unabhängig davon, ob das Universum endlich oder unendlich ist. Die Gültigkeit des kosmologischen Prinzips muß man wohl hinnehmen. Man kann das guten Gewissens tun angesichts des Erfolgs des Lambda-CDM Modells. Deshalb erfolgt nach der ART der "große Zusammensturz" überall. Worauf ja auch 'Ich' schon hingewiesen hat. Nach Newton gibt's bei einem Zusammensturz ein Zentrum. Der Raum merkt davon nichts.

Hallo Timm,

gerade lese ich im neuen Heft "Spektrum der Wissenschaft auf Seite 64, das die Energiedichte im Universum sich wie folgt zusammensetzt:

68,3% dunkle Energie
26,8% dunkle Materie
4,9% Baryonische Materie

Der Autor sagt aber nichts darüber, ob das alles zusammen ausreicht, um das Universum in sich selbst zurückzuschließen, so dass es zu jedem Zeitpunkt ein endliches 3-D-Volumen hat.

Hat jemand darüber Informationen?

M.f.G. Eugen Bauhof

Timm
23.08.13, 15:49
Hallo Eugen,

Seite 62 rechts wird nebenbei erwähnt, "daß der Raum auf großen Skalen flach ist". Wenn das so ist, gibt es keinen "großen Zusammensturz", sondern ewige Expansion. Falls der w Parameter (Seite 69) < -1 ist, droht dem Universum der Big Rip (Atome werden zerrissen). Darauf deutet aber nach den aktuellen Daten nichts hin.

Gruß, Timm

JoAx
01.09.13, 11:24
Hallo ICH,
ich dachte/denke nicht, dass ich dem Newtonschen Weltbild (allzu sehr) verhaftet bin. Für mich wäre/ist die Donut bzw. Sphärenform einfach die natürliche Form der „Raumzeitmaterie“Verteilung. Das "2p-1/+1 Orbitalmodell" (http://commons.wikimedia.org/wiki/File%3AHydrogen_eigenstate_n2_l1_m-1.png) des „Gravitationsfelds“ in einem n-Dimensionalen Universum. Die „Aufenthaltswahrscheinlichkeit der Materie “in einem n-Dimensionalen Universum in seinem „Feld“.


Alex, ich kapiere nicht, wie du so etwas als Antwort auf Ich's Ausführungen produzieren kannst. :confused:

Eyk van Bommel
10.09.13, 16:30
Alex, ich kapiere nicht, wie du so etwas als Antwort auf Ich's Ausführungen produzieren kannst. :confused:

(zurück vom Urlaub)

Hallo JoAx,
gute Frage – manchmal gehen mit mir die…:o

Aber ich erhalte als Antwort ja häufiger, dass ich der newtonschen Welt angehöre. Dabei bin ich ja weiter von Newton weg wie…

Aber bis auf die Form des Donuts und der Tatsache, dass man imho die Form des Universums so natürlich beschreiben werden kann wie die eines Orbitals sollte dabei nicht zur Aussage kommen.

Ich wollte nur zum Ausdruck bringen, dass die Form des Universums nicht von außen vorgegeben wird.

Nur habe ich die bisherigen Diskussionen nicht so verstanden – die eigentliche Form des Universums kann bisher nicht vorhergesagt werden. Es gibt verschiedene Topologien zur Auswahl und diese hängen ja nicht allein von der Menge bzw. der Eigenschaft der „Raumzeit-Materie“ ab.:rolleyes: Die Menge bzw. der Eigenschaft der „Raumzeit-Materie“ ändert nur was an der Krümmung.

Wie man vorhersagen kann, wie sich die Form des Universums in Abhängigkeit der Materiendichte-/Verteilung verhält?

In einem Universum das aus Kaffeebohnen besteht und genügend Masse besitzt um zu einem SL zu kollabieren, würde nach meinem Verständnis die Raumzeit außerhalb des SL nach dem „Kollaps“ bestehen bleiben und sich zudem mit c ausbreiten. "Die Form bleibt erhalten"

Gruß
EvB

Bauhof
11.09.13, 11:21
Wie man vorhersagen kann, wie sich die Form des Universums in Abhängigkeit der Materiendichte-/Verteilung verhält?

Hallo Eyk,

Die Materiendichte-/Verteilung ist gegeben durch die dunkle Materie, die sichtbare Materie, die dunkle Energie und wird durch die ART beschrieben.

In einem Universum das aus Kaffeebohnen besteht und genügend Masse besitzt um zu einem SL zu kollabieren, würde nach meinem Verständnis die Raumzeit außerhalb des SL nach dem „Kollaps“ bestehen bleiben und sich zudem mit c ausbreiten. "Die Form bleibt erhalten"

Gemäß der ART wirkt die Masse auf die Raumzeit zurück und auch umgekehrt. Wenn die gesamte Universum-Masse kollabiert, dann hat dies auch Rückwirkungen auf die Raumzeit. Sie kann deshalb nicht so bestehen bleiben wie vor dem Kollaps.

Und warum sollte sich die Raumzeit nach dem Kollaps mit c ausbreiten?

M.f.G. Eugen Bauhof

Timm
11.09.13, 18:02
Hallo Eyk,


Wie man vorhersagen kann, wie sich die Form des Universums in Abhängigkeit der Materiendichte-/Verteilung verhält?

In einem Universum das aus Kaffeebohnen besteht und genügend Masse besitzt um zu einem SL zu kollabieren, würde nach meinem Verständnis die Raumzeit außerhalb des SL nach dem „Kollaps“ bestehen bleiben und sich zudem mit c ausbreiten. "Die Form bleibt erhalten"


das Verhältnis Energiedichte zu kritischer Dichte liefert die lokale Geometrie, nicht die Topologie des Universums. Indizien für Letztere könnten im CMB verborgen sein. Mehr läßt sich derzeit dazu kaum sagen.

Die Daten betätigen das Lambda-CDM Modell und damit auch das zugrunde liegende FLRW-Modell. Wenn das so ist, kann das Universum nicht zu einem Schwarzen Loch kollabieren. Im Rahmen einer globalen FRW-Metrik können allenfalls lokal Schwarze Löcher entstehen, was ja auch der Fall ist. Oder anders gesagt, FRW-Metrik kann nicht gleichzeitig global Schwarzschild Metrik sein.

Gruß, Timm

Eyk van Bommel
11.09.13, 21:12
Grundsätzlich ist mir das (meiste) bekannt. Aber gleichzeitig scheine ich von wenigen Dingen weniger Ahnung zu haben wie von der Topologie und der inneren/äußeren Raumkrümmung.

Ich will mein Problem mal folgendermaßen beschreiben.

Wir nehmen ein SL und eine Masse X die das SL (homogen verteilt) umkreist (nahe Ereignishorizont). Hier könnte die Masse fast beliebig groß werden ohne, dass die Massen zusammenstürzen? Es würden sich max. lokale SL’s bilden die das „große“ SL umkreisen – aber nur schwer ein einzelnes.

Ich meine nahe am EH kann man ja fast seinen eigenen Hintern (auf einer Geodäte) wieder sehen und man müsste sich fragen warum stürzt nicht alles zusammen - Die (sichtbare) Masse sollte ausreichen.

Meine Frage zielt darauf Hinaus: Kann die „kritische Masse“ nicht größer sein als „erlaubt“ ohne dass es zum Kollaps kommt, da (wie oben) die Topologie (äußere Krümmung) es einfach nicht erlaubt.

Gruß
EvB
@Timm
Die Daten betätigen das Lambda-CDM Modell und damit auch das zugrunde liegende FLRW-Modell.
Das beschreibt aber ggf. nur die innere Krümmung?
@Bauhof
Die Materiendichte-/Verteilung ist gegeben durch die dunkle Materie, die sichtbare Materie, die dunkle Energie und wird durch die ART beschrieben.
Innere Krümmung?

Bauhof
12.09.13, 17:25
Grundsätzlich ist mir das (meiste) bekannt. Aber gleichzeitig scheine ich von wenigen Dingen weniger Ahnung zu haben wie von der Topologie und der inneren/äußeren Raumkrümmung.

Hallo Eyk,

ich weiß jetzt zwar nicht, was dein Problem mit der inneren/äußeren Raumkrümmung zu tun hat. Aber hier findet man bei Wikipedia (http://de.wikipedia.org/wiki/Raumkr%C3%BCmmung#Innere_und_.C3.A4u.C3.9Fere_Kr.C 3.BCmmung) die Erklärungen:

Man unterscheidet bei der Krümmung zwischen der inneren und der äußeren Krümmung.

Die innere Krümmung lässt sich anhand der Geometrie im gekrümmten Raum selbst feststellen. Beispielsweise haben Dreiecke auf der Kugeloberfläche eine Innenwinkelsumme von mehr als 180°, im Gegensatz zu ebenen Dreiecken mit einer konstanten Winkelsumme von 180°. Die innere Krümmung kann positiv sein (wie auf einer Kugel) oder negativ (wie beim Kühlturm eines AKWs). In einem negativ gekrümmten Raum ist die Innenwinkelsumme kleiner als 180°.

Die äußere Krümmung kann nur festgestellt werden, indem die Lage des Raums im umgebenden, höherdimensionalen Raum, die so genannte Einbettung, betrachtet wird. Flächen mit äußerer Krümmung, aber ohne innere Krümmung erhält man z. B., indem man ein Blatt Papier aufrollt, wellt, oder sonst wie verbiegt, ohne dass man es entweder zerreißt oder verknittert. Auf solchen Flächen ändern sich die Gesetze der Geometrie nicht (Beispiel: Die Innenwinkelsumme eines aufs Papier gemalten Dreieck ändert sich nicht, wenn man das Papier aufrollt).

Eindimensionale Räume (Linien) haben grundsätzlich keine innere Krümmung, sondern nur, sofern sie in einen höherdimensionalen Raum eingebettet sind, eine äußere Krümmung.

M.f.G. Eugen Bauhof

P.S.
Du drückst dich so unklar und verworren aus, so dass man wahrscheinlich dauernd aneinander vorbeiredet. Sag doch klar, welche Dinge dir bekannt und welche dir unbekannt sind.

Eyk van Bommel
12.09.13, 19:17
P.S.
Du drückst dich so unklar und verworren aus, so dass man wahrscheinlich dauernd aneinander vorbeiredet. Sag doch klar, welche Dinge dir bekannt und welche dir unbekannt sind.

Du kannst dir sicher sein, wenn ich es klarer formulieren könnte würde ich es (jedes Mal) tun.

Manchmal benötigt ich aber 2-3 Antworten um meine Frage richtig zu formulieren. Damit aus einem Gefühl ein klarer Gedanke wird.

Aber nun bekomme ich es hin (Licht ging an):

Kann die äußere Krümmung der Inneren entgegenwirken.

Gruß
EvB

Bauhof
13.09.13, 09:56
Aber nun bekomme ich es hin (Licht ging an):
Kann die äußere Krümmung der Inneren entgegenwirken.

Hallo Eyk,

ich befürchte, dein Licht ging nicht an, sondern aus. Ich habe doch Wikipedia zitiert:

Wenn du ein Dreieck auf ein Blatt Papier zeichnest, dann beträgt die Innenwinkelsumme 180⁰. Wenn das Papier gerollt wird, ändert sich die Innenwinkelsumme des Dreiecks nicht. Also kann die hinzugekommene äußere Krümmung (hervorgerufen durch das Aufrollen) der inneren Krümmung nicht entgegen gewirkt haben, weil die Winkelsumme im Dreieck gleich geblieben ist.

M.f.G. Eugen Bauhof

Eyk van Bommel
13.09.13, 10:45
In diesem Beispiel ist die Sachlage klar.

Aber Raumzeit und Papier sind ja nur bedingt ähnlich. So wie Gummiband oder -tuch. Gibt es keine Beispiele in dem dies nicht so wäre?

Noch einmal an einem Beispiel. Ich male zwei Punkte (Punktmassen) auf ein Papier. Jedem ist klar, dass die Punkte sich nähern würden (endliches-offenes Universum).
Nun bilde ich einen Zylinder und klebe die Enden so zusammen, dass der Abstand der beiden Punkte in beide Richtungen gleichweit ist. Jetzt würde es zu keiner Anziehung mehr kommen.

Gruß
EvB

Bauhof
13.09.13, 15:33
Noch einmal an einem Beispiel. Ich male zwei Punkte (Punktmassen) auf ein Papier. Jedem ist klar, dass die Punkte sich nähern würden (endliches-offenes Universum).
Nun bilde ich einen Zylinder und klebe die Enden so zusammen, dass der Abstand der beiden Punkte in beide Richtungen gleichweit ist. Jetzt würde es zu keiner Anziehung mehr kommen.

Hallo Eyk,

doch, die "Anziehung" (wenn man bei Newton bleibt) ist nach wie vor da. Aber nachdem die Punktmassen in deinem Zylinderuniversum sowohl nach links und auch nach rechts den gleichen Abstand voneinander haben, heben sich die Anziehungen rein theoretisch gegenseitig auf.

Aber bei der kleinsten Schwankung wird die Anziehung nach links oder nach rechts größer und die zwei Punktmassen bewegen sich aufeinander zu.

Mir ist nicht klar, auf was du dem Beispiel hinauswillst. Was willst du uns damit sagen?

M.f.G. Eugen Bauhof

Eyk van Bommel
13.09.13, 16:31
Hallo Eugen,
Mir ist nicht klar, auf was du dem Beispiel hinauswillst. Was willst du uns damit sagen?

Ich will nichts Aussagen und das Ziel ist für mich dasselbe wie im Eingangsthread.

Erst einmal bin ich froh, dass diese Annahme stimmt :)

2. Hier hat die äußere Krümmung ja Einfluss auf die Innere? Indirekt

Hmm – und ändert sich nicht doch auch mit dem „Kurzschluss“ mit dem Moment des „Verklebens“ die innere Krümmung? :rolleyes:

3. In einem perfekt symmetrischen Universum – wäre der Zustand stabil. Bzw. hängt es ja sehr vom Radius des Zylinders ab.

4. Hier sind wir bei der Anfangsfrage. Wenn die Punkte (alle Punkte) sich nähern und zu einem SL werden ist mir einfach immer noch nicht klar, warum hier das „Gravitationsfeld“ das sich über die Zylinderfläche verteilt verschwinden sollte. Da muss das Feld irgendwo "reißen" was aber

Gruß
EvB

Timm
14.09.13, 09:55
Eyk, Dein Problem besteht in der vollständigen Negierung aller Versuche, Dir auf die Sprünge zu helfen.

4. Hier sind wir bei der Anfangsfrage. Wenn die Punkte (alle Punkte) sich nähern und zu einem SL werden ist mir einfach immer noch nicht klar, warum hier das „Gravitationsfeld“ das sich über die Zylinderfläche verteilt verschwinden sollte. Da muss das Feld irgendwo "reißen" was aber

Diese Vorstellung ist falsch. Jetzt bist Du - indem Du ein Massenzentrum (SL) annimmst - wieder bei Schwarzschild oder Newton. Im Kontext Verkleben/Topologie gilt jedoch das Kosmologische Prinzip, also kein Massenzentrum. Also weder Schwarzschild noch Newton.

Gruß, Timm

Timm
14.09.13, 10:08
Hallo Eugen,

doch, die "Anziehung" (wenn man bei Newton bleibt) ist nach wie vor da. Aber nachdem die Punktmassen in deinem Zylinderuniversum sowohl nach links und auch nach rechts den gleichen Abstand voneinander haben, heben sich die Anziehungen rein theoretisch gegenseitig auf.

Aber bei der kleinsten Schwankung wird die Anziehung nach links oder nach rechts größer und die zwei Punktmassen bewegen sich aufeinander zu.

hast Du eine Idee, wie angewandt auf ein Zylinderuniversum "wenn man bei Newton bleibt" zu rechtfertigen wäre?

Meines Wissens ist der Newton'sche Raum nicht gekrümmt. Punktmassen bewegen sich immer aufeinander zu und Anziehungen heben sich nie auf.

Gruß, Timm

Eyk van Bommel
14.09.13, 12:53
@Timm
Ich negiere dass ich negiere. Das kosmologische Prinzip ist für mich ein wichtiges Prinzip dessen Existenz jedoch ein geschlossenes Universum verlangt?

Und ich habe deine Antwort sehr wohl verstanden. Das würde aber bedeuten (nach meiner Vorstellung) dass der Radius des Zylinders kleiner wird? Oder?

Gruß
EvB

Bauhof
14.09.13, 13:26
Das kosmologische Prinzip ist für mich ein wichtiges Prinzip dessen Existenz jedoch ein geschlossenes Universum verlangt?

Hallo Eyk,

war das eine Frage oder eine Feststellung?

M.f.G. Eugen Bauhof

Bauhof
14.09.13, 13:44
Hallo Eugen, hast Du eine Idee, wie angewandt auf ein Zylinderuniversum "wenn man bei Newton bleibt" zu rechtfertigen wäre?

Meines Wissens ist der Newton'sche Raum nicht gekrümmt. Punktmassen bewegen sich immer aufeinander zu und Anziehungen heben sich nie auf.
Gruß, Timm

Hallo Timm,

ja, der Newtonsche Raum hat keine innere Krümmung im Gegensatz zur Einsteinschen Raumzeit. Möglicherweise hat der Newtonsche Raum eine äußere Krümmung, so wie sich das Eyk vermutlich mit seinem gerollten Zylinder vorstellt.

Auf der Zylinderoberfläche ist die euklidische Geometrie anwendbar, weil sie keine innere Krümmung aufweist.

M.f.G. Eugen Bauhof

Bauhof
14.09.13, 13:52
2. Hier hat die äußere Krümmung ja Einfluss auf die Innere? Indirekt

Hallo Eyk,

die äußere Krümmung hat m.E. keinen Einfluss auf die innere Krümmung, weder direkt noch indirekt. Wie auch?

M.f.G. Eugen Bauhof

Timm
14.09.13, 15:22
@Timm
Ich negiere dass ich negiere. Das kosmologische Prinzip ist für mich ein wichtiges Prinzip dessen Existenz jedoch ein geschlossenes Universum verlangt?
Das KP ist eine plausible und vereinfachende Annahme ohne die die Einsteinschen Feldgleichungen kaum lösbar sind. Es gilt für alle darauf beruhenden kosmologischen Modelle. Das Universum kann je nach Parametern offen oder geschlossen sein. Davon hängt diese Unterscheidung ab, nicht vom per se gültigen KP.
Die Lösungen der Einsteinschen Gleichungen lassen die Frage nach der Topologie offen. Die Dynamik des Universums ist deshalb unabhängig von dessen Topologie. Daß das KP immer gilt und damit unabhängig davon welche Topologie das Universum hat, dürfte klar sein.

Und ich habe deine Antwort sehr wohl verstanden. Das würde aber bedeuten (nach meiner Vorstellung) dass der Radius des Zylinders kleiner wird? Oder?

Nein, die Entfernungen zwischen mitbewegten Beobachtern wachsen oder schrumpfen je nach der Dynamik des Universums, s.o. Das ist so, ob nun 3-Torus, 3-Zylinder oder 3-Ebene, was auch immer.
Die Topologie sagt nach meiner Kenntnis nur was aus, ob und auf welchen Pfaden Licht prinzipiell zum Ausgangspunkt zurück laufen kann.

Gruss, Timm

Eyk van Bommel
14.09.13, 16:57
@Eugen
die äußere Krümmung hat m.E. keinen Einfluss auf die innere Krümmung, weder direkt noch indirekt. Wie auch?
Lass mich es anders ausdrücken. Durch das Verkleben kann man einen Kollaps verhindern der ohne Verklebung unvermeidlich ist.

Zudem würde sich z.b eine Gravitationswelle sich eine beide Richtungen ausbreiten. Das Universum sieht geschlossen jedenfalls anders aus.

@Timm
Danke für deine verständlichen Aussagen und solange keiner was dagegen schreibt nehme ich deine Aussagen als richtig an.

Wie kann es in einem offen Universum eine KP "am Rand" geben?

Wie können sich alle Objekte in einem Zylinder nähern ohne den Radius dessen zu ändern?

Gruß
EvB

PS: Schreibe vom Handy hoffe, dass dies mein geschreibsel nicht weiter verschlimmmert.

Timm
14.09.13, 18:00
Wie kann es in einem offen Universum eine KP "am Rand" geben?

Wie können sich alle Objekte in einem Zylinder nähern ohne den Radius dessen zu ändern?


Das Kosmologische Prinzip beinhaltet ja gerade, daß es keinen Rand gibt.

Du läßt Dich vielleicht in die Irre führen, indem Du die 2-Analogien zu wörtlich nimmst. Die Vorstellung auf den Zylinder zu schauen ist unnötig und verwirrt eher. Es gibt keine theoretische Aussage, wonach das Universum in eine höhere Mannigfaltigkeit eingebettet ist.

Stell Dir vor, die Oberfläche des Zylinders (Torus, ...) sei der Raum. Ich kann es auch nur laienhaft ausdrücken und lasse mich gern verbessern: Der Umfang/Radius des Zylinders ergibt sich letztlich aus der Laufzeit des Lichts zurück zum Ausgangspunkt. Dabei ist u/r abhängig von der Krümmung, also nicht 2 pi. Wenn der 3-Zylinder kollabiert, schrumpfen alle Distanzen einschließlich der durch die erwähnte Lichtlaufzeit gegebenen. Bei alledem muß man allerdings im Auge behalten, daß im expandierenden, bzw. Kontrahierenden Universum Distanzen nicht eindeutig definiert sind.

Gruss, Timm

Eyk van Bommel
14.09.13, 20:47
Das Kosmologische Prinzip beinhaltet ja gerade, daß es keinen Rand gibt.
Dann habe ich es falsch verstanden.

Universum offen - Universum mit Rand
Universum geschlossen - kein Rand

Gruß
EvB

Marco Polo
14.09.13, 21:21
Dann habe ich es falsch verstanden.

Universum offen - Universum mit Rand
Universum geschlossen - kein Rand

Das siehst du falsch, Eyk.

Die Raumzeit ist nicht in einen höherdimensionalen Raum eingebettet.

Aktuell wird von einer flachen Raumzeit mit euklidischer Geometrie ausgegangen.

Ob offen oder geschlossen. Es gibt keinen Rand.

Nimm als Analogon die Kugeloberfläche.

Hat sie einen Rand? Nein.

Hat sie einen Mittelpunkt? Nein.

Ist sie endlich? Nein.

Sie hat aber einen endlichen Wert für ihre Oberfläche.

Gr., MP

Eyk van Bommel
14.09.13, 23:04
Sorry - ich habe da wohl einmal was missverstanden und seitdem immer falsch gelesen. Geschlossen bezieht sich auf die Endlichkeit des Universums und zwar zeitlich und nicht räumlich.

Ersetzt den vorherigen Antworten "offen" mit Universum mit Rand.

:o

@Timm
Nach deiner letzten Antwort muss ich davon ausgehen, dass es kein (mir) vorstellbares Modell gibt, dass dieses Bild wiedergibt? Es erscheint mir fast so also würde man eine Hypersphäre /Zylinder erzeugen und dann wieder platt drücken bis r=0 bzw. die zusätzliche Dimension wieder verschwunden ist.

Bei allem was ich nun gelernt habe, Frage ich mich schon welche Bedeutung die Topologie des Universums für uns hat, wenn es keine Auswirkung auf uns hat?

Gruß
EvB

Bauhof
15.09.13, 09:49
Die Raumzeit ist nicht in einen höherdimensionalen Raum eingebettet.

Hallo Marc,

da wäre ich vorsichtig. Wir wissen es nicht, ob die Raumzeit in einem höherdimensionalen Raum eingebettet ist oder nicht. Stephen Hawking z.B. hat die Raumzeit in seiner kosmologischen Theorie in einem höherdimensionalen Raum eingebettet.

Nimm als Analogon die Kugeloberfläche.
Hat sie einen Rand? Nein.
Hat sie einen Mittelpunkt? Nein.
Ist sie endlich? Nein.
Sie hat aber einen endlichen Wert für ihre Oberfläche.

Der Kugeloberfläche gesteht man m.E. einen endlichen Wert zu, weil der Inhalt der Kugeloberfläche einen endlichen Wert besitzt. Vermutlich meinst du, dass man die Kugeloberfläche unendlich oft umrunden kann, ohne an eine Grenze zu stoßen. Dann könnte man die Kugeloberfläche in einem gewissen Sinne als unendlich bezeichnen. Aber sie ist nur unbegrenzt.

M.f.G. Eugen Bauhof

Marco Polo
15.09.13, 17:04
Der Kugeloberfläche gesteht man m.E. einen endlichen Wert zu, weil der Inhalt der Kugeloberfläche einen endlichen Wert besitzt. Vermutlich meinst du, dass man die Kugeloberfläche unendlich oft umrunden kann, ohne an eine Grenze zu stoßen. Dann könnte man die Kugeloberfläche in einem gewissen Sinne als unendlich bezeichnen. Aber sie ist nur unbegrenzt.


Genau Eugen. Unbegrenzt passt besser.

Grüsse, MP

Jogi
15.09.13, 17:16
Bei allem was ich nun gelernt habe, Frage ich mich schon welche Bedeutung die Topologie des Universums für uns hat, wenn es keine Auswirkung auf uns hat?

Keine.
Soweit war ich auch schon...;)

Meine 2Cents hierzu:
Die Diskussion um die Topologie des Universums hat einzig auf die Darstellung des Universums in diversen Modellen Einfluss.

Und:
Jede modellhafte Darstellung ist eben nur eine modellhafte Darstellung, aber mehr können wir nicht leisten, damit muß man sich abfinden.

Eyk van Bommel
16.09.13, 09:56
Hallo Jogi,
Die Diskussion um die Topologie des Universums hat einzig auf die Darstellung des Universums in diversen Modellen Einfluss.
Offenbar ist selbst die Darstellung dann immer falsch. Bzw . gibt es keine Darstellung die Topologie tatsächlich wiederspiegelt.

Und wenn ich n-Topologien annehmen kann – die sich physikalisch alle nicht Unterscheiden, dann ist das keine Physik sondern reine Mathematik.

Ich würde gerne noch einmal auf die zwei Massenpunkte auf einem Blatt Papier eingehen.
Ohne äußere Krümmung würden die beiden Punkte sich nähern. Wenn die Topologie des Universums eine Zylinderform hätte und der Abstand der Punkte zufällig passt, dann würden die Punkte sich nicht nähern – bei Newton.
[BTW: Dieses Bild darzustellen war mein Versuch mit dem SL in der Mitte, dass das Zylinderinnere (SL bis EH) wiederspiegeln sollte. ]

Im Friedman/Einstein Universum hingegen wahrscheinlich doch bzw. trotzdem????

Frage: Gibt es im "Friedman/Einstein"-Universum keine Topologie die das zusammenziehen der Massen ähnlich beeinflusst wie die Zylinderform bei Newton?

Gruß
EvB

Timm
16.09.13, 10:42
Hallo Jogi,

Offenbar ist selbst die Darstellung dann immer falsch. Bzw . gibt es keine Darstellung die Topologie tatsächlich wiederspiegelt.

Und wenn ich n-Topologien annehmen kann – die sich physikalisch alle nicht Unterscheiden, dann ist das keine Physik sondern reine Mathematik.

Ich würde gerne noch einmal auf die zwei Massenpunkte auf einem Blatt Papier eingehen.
Ohne äußere Krümmung würden die beiden Punkte sich nähern. Wenn die Topologie des Universums eine Zylinderform hätte und der Abstand der Punkte zufällig passt, dann würden die Punkte sich nicht nähern – bei Newton.
[BTW: Dieses Bild darzustellen war mein Versuch mit dem SL in der Mitte, dass das Zylinderinnere (SL bis EH) wiederspiegeln sollte. ]

Im Friedman/Einstein Universum hingegen wahrscheinlich doch bzw. trotzdem????

Frage: Gibt es im "Friedman/Einstein"-Universum keine Topologie die das zusammenziehen der Massen ähnlich beeinflusst wie die Zylinderform bei Newton?

Ich weiß nicht, was Jogi mit "Darstellung des Universums in diversen Modellen" wirklich meint. Wie schon erwähnt, machen die auf den Einstein'schen Gleichungen beruhenden Modelle keinerlei Aussagen über die Topologie des Universums. Vielmehr ist je nach derTopologie des Universums dieses endlich oder unendlich und da sprechen wir über Physik. Prinzipiell könnten der CMB Hinweise liefern, der 3-Torus wird ja diskutiert.

Den Newton'schen Zylinder halte ich wie schon weiter oben erwähnt für ein sich selbst widersprechendes Gedankenspiel. Ich kann Dir nur raten davon zu lassen, weil es Dich unnötig verwirrt und Dich stattdessen in das FRW Modell einzulesen. Im FRW 3-Zylinder gibt es keine 2 Massenpunkte, die nur dann, wenn sie einander exakt gegenüber stehen nicht aufeinander zu fallen. Du kannst Dir die Massenpunkte (Kaffeebohnen) als mitbewegt im Hubble Fluss vorstellen. Von jedem beliebigen Punkt aus betrachtet nehmen die Entfernungen ringsum zu (bzw. ab). Das ist so beim 3-Zylinder, der 3-Ebene usw., also völlig unabhängig von der Topologie.

Gruß, Timm

Jogi
16.09.13, 17:40
Ich weiß nicht, was Jogi mit "Darstellung des Universums in diversen Modellen" wirklich meint.
Ich meine, daß jedes topologische Modell nur ein Hilfskonstrukt sein kann, aber niemals eine realitätsgetreue Abbildung des Universums.

Ich hab's ja schon mal gesagt:

Statische Modelle, die von einer Randlosigkeit ausgehen, müssen sich einer exotischen Topologie bedienen.

Modelle, die die Expansion als Dynamik beinhalten, brauchen das gar nicht,
Da genügt die sphärische Form in Verbindung mit der Grenzgeschwindigkeit c für jede Information, die uns erreicht.
Ich hebe das deshalb hervor, da diese Modelle durchaus ein Universum zulassen, in dem uns eben nicht alle Informationen erreichen.
Das was wir als "Rand" interpretieren könnten, ist lediglich die Informationsgrenze für uns als Beobachter.

Und, um auf den Threadtitel Bezug zu nehmen, ja, auch für die Gravitation ist diese Grenze von Bedeutung.
Die Gravitation unserer Galaxie wirkt nicht über diese Grenze hinaus.

JoAx
16.09.13, 17:40
Und wenn ich n-Topologien annehmen kann – die sich physikalisch alle nicht Unterscheiden, dann ist das keine Physik sondern reine Mathematik.


Diese "reine Mathematik" zeigt auf, welche Möglichkeiten es überhaupt geben kann. Das ist doch gut und positiv. Dass man zwischen diesen an Hand der experimentellen Daten u.U. nicht unterscheiden kann ... du weisst ja - das Leben ist kein Wunschkonzert.


Ich würde gerne noch einmal auf die zwei Massenpunkte auf einem Blatt Papier eingehen.


Als erstes musst du dir darüber klar werden, dass ein s.g. 'geschlossenes Universum' nicht räumlich geschlossen bedeutet, sondern zeitlich - es endet im Big Crunch. Dann, dass ein räumlich geschlossenes Universum (mit einer kompakten räumlichen Topologie) auch offen sein kann - die Expansion endet nie.

Was willst du nun betrachten?
Räumlich kompakte Topologie? (= endlos aber nicht unendlich)


dann würden die Punkte sich nicht nähern – bei Newton.


Bei "Newton" würde sich an dem Raum nie etwas ändern. Einfach so.


Grüße

JoAx
17.09.13, 02:49
Hi, Jogi!

Ich meine, daß jedes topologische Modell nur ein Hilfskonstrukt sein kann, aber niemals eine realitätsgetreue Abbildung des Universums.


Ich weiss nicht, lässt sich das nicht über jedes unserer Modelle sagen? Eine bestimmte Topologie wäre halt eine Eigenschaft unseres Universums.


Statische Modelle, die von einer Randlosigkeit ausgehen, müssen sich einer exotischen Topologie bedienen.


Ich denke nicht, dass man statische Modelle betrachtet. Egal wie - die Raumzeit ist ein dynamisches Objekt.


Da genügt die sphärische Form


Eine 3-Sphäre ist aber auch keine triviale Topologie.


Das was wir als "Rand" interpretieren könnten, ist lediglich die Informationsgrenze für uns als Beobachter.


Genau. Ich würde das nicht Mal als "Rand" bezeichnen. Ein Rand ist das, was bsw. ein Blatt Papier hat. Ein Universum "darf" so etwas schon aus dem kosmologischen Prinzip nicht haben. :D


Grüße.

Timm
17.09.13, 09:57
Hallo Jogi, damit Du besser ausgelastet bist, kommt jetzt auch noch mein Senf dazu.

Statische Modelle, die von einer Randlosigkeit ausgehen, müssen sich einer exotischen Topologie bedienen.

Modelle, die die Expansion als Dynamik beinhalten, brauchen das gar nicht,
Da genügt die sphärische Form in Verbindung mit der Grenzgeschwindigkeit c für jede Information, die uns erreicht.
Ich hebe das deshalb hervor, da diese Modelle durchaus ein Universum zulassen, in dem uns eben nicht alle Informationen erreichen.
Das was wir als "Rand" interpretieren könnten, ist lediglich die Informationsgrenze für uns als Beobachter.

Ich glaube, Du erliegst hier einem Irrtum. Alle Einsteinschen Modelle sind randlos, weil sie auf dem Kosmologischen Prinzip beruhen. Kein solches Modell muß sich irgendeiner Topologie bedienen. Die Topologie ist von Anbeginn an wie sie ist und es gibt keinerlei Zusammenhang zwischen der Topologie (ob exotisch, trivial, endlich oder unendlich) und der Entwicklung (statisch/dynamisch, Expansion/Kontraktion) des Universums.

Und wenn Du "Rand" mit "Information erreichen" verknüpfst, meint Du nicht Rand im oben erwähnten Sinn, sondern Horizont.

Nicht-Einsteinsche Modelle werden in der Kosmologie kaum diskutiert. Eher schon die Möglichkeit, daß das kosmologische Prinzip doch nicht gilt. Denn klar - obwohl das Standardmodell ausgezeichnet zu den Daten passt (SN Ia und CMB) - werden wir vermutlich nie letzte Sicherheit haben, daß das Universum überall gleich aussieht.

Gruß, Timm

JoAx
17.09.13, 11:22
Hi, Timm!


Ich glaube, Du erliegst hier einem Irrtum. Alle Einsteinschen Modelle sind randlos, weil sie auf dem Kosmologischen Prinzip beruhen. Kein solches Modell muß sich irgendeiner Topologie bedienen. Die Topologie ist von Anbeginn an wie sie ist und es gibt keinerlei Zusammenhang zwischen der Topologie (ob exotisch, trivial, endlich oder unendlich) und der Entwicklung (statisch/dynamisch, Expansion/Kontraktion) des Universums.

Und wenn Du "Rand" mit "Information erreichen" verknüpfst, meint Du nicht Rand im oben erwähnten Sinn, sondern Horizont.

Nicht-Einsteinsche Modelle werden in der Kosmologie kaum diskutiert. Eher schon die Möglichkeit, das das kosmologische Prinzip doch nicht gilt. Denn klar - obwohl das Standardmodell ausgezeichnet zu den Daten passt (SN Ia und CMB) - werden wir vermutlich nie letzte Sicherheit haben, daß das Universum überall gleich aussieht.


Ich will da auch paar Gedanken los werden. Mal sehen ob es passt.

Die Topologie und Geometrie können zwar von einander getrennt betrachtet werden, es passt aber nicht jede Geometrie zu jeder Topologie. Eine 3-Sphäre würde nicht zu einem flachen Raum passen. Sie sind also nicht völlig zusammenhanglos, aber zur selben Geometrie können mehrere Topologien passen. Insofern kann die Geometrie nicht alles über die Topologie aussagen. Man muss da andere "Effekte" suchen.

Das kosmologische Prinzip kann im Prinzip (:o) auch überprüft werden, nur muss dazu das Universum in Gänze überblickbar sein => kompakt und klein genug, dass unsere Sichtweite ausreicht, es auch zu erkennen.

Ansonsten kann man das kosmologische Prinzip auch so verstanden werden - wenn wir bsw. ein "Rohr-Universum" feststellen, dann tuen das auch alle anderen.


Grüße, Johann

Timm
17.09.13, 13:47
Hi Johann,

Die Topologie und Geometrie können zwar von einander getrennt betrachtet werden, es passt aber nicht jede Geometrie zu jeder Topologie. Eine 3-Sphäre würde nicht zu einem flachen Raum passen. Sie sind also nicht völlig zusammenhanglos, aber zur selben Geometrie können mehrere Topologien passen. Insofern kann die Geometrie nicht alles über die Topologie aussagen. Man muss da andere "Effekte" suchen.

Das kosmologische Prinzip kann im Prinzip (:o) auch überprüft werden, nur muss dazu das Universum in Gänze überblickbar sein => kompakt und klein genug, dass unsere Sichtweite ausreicht, es auch zu erkennen.

Ansonsten kann man das kosmologische Prinzip auch so verstanden werden - wenn wir bsw. ein "Rohr-Universum" feststellen, dann tuen das auch alle anderen.

Ich stimme Dir da zu. Vielleicht noch ergänzend, Topologie wird ja oft mit "Gestalt des Universums" identifiziert. Gemeint ist aber ein Raum konstanter Krümmung, was ja aus dem KP folgt. Wenn der Raum bei uns flach ist, wie es scheint, muß er überall flach sein. Insofern verliert der Begriff Topologie bei einer davon abweichenden, zb. "verbeulten" Gestalt des Universums seine Bedeutung.

Gruß, Timm

JoAx
17.09.13, 14:13
Gemeint ist aber ein Raum konstanter Krümmung, was ja aus dem KP folgt. Wenn der Raum bei uns flach ist, wie es scheint, muß er überall flach sein.


Ich denke, hier ist es an der Zeit, die Genauigkeit der Messdaten ins Gespräch zu bringen. Man kann aus diesen (noch) nicht sagen, dass der Raum exakt flach ist. Nur, dass es ziemlich flach ist. Aber so, dass es immer noch auch leicht positive oder auch leicht negative Krümmung erlaubt.

Wie ist es beim nicht-flachen Torus? Wenn unser Sichtfeld klein genug wäre, dann müssten doch abhängig von der Position auf diesem, entweder

a. positive Krümmung, oder
b. negative Krümmung, oder gar
c. auf der einen Seite positive und in der entgegengesetzter negative Krümmung zu messen sein.

Oder?


Insofern verliert der Begriff Topologie bei einer davon abweichenden, zb. "verbeulten" Gestalt des Universums seine Bedeutung.


Das würde ich nicht sagen. Eine verbeulte Sphäre ist topologisch immer noch eine Sphäre. So, wie eine Tasse topologisch immer noch ein Torus ist. So etwas würde die Aufgabe aber erschweren, die Topologie zu entziffern. Aber vlt. kommt man irgendwann so weit, auch solche Modelle zu betrachten, wenn die Computer noch etwas leistungsfähiger geworden sind. :D


Grüße

Timm
17.09.13, 15:50
Ich denke, hier ist es an der Zeit, die Genauigkeit der Messdaten ins Gespräch zu bringen. Man kann aus diesen (noch) nicht sagen, dass der Raum exakt flach ist. Nur, dass es ziemlich flach ist. Aber so, dass es immer noch auch leicht positive oder auch leicht negative Krümmung erlaubt.
Genau, wobei es die Kosmologen für wenig wahrscheinlich halten, daß Omega knapp neben 1 liegt. Aber ausschließen kann man das natürlich nicht.

Wie ist es beim nicht-flachen Torus?
Laß Dich nicht täuschen indem Du ihn als Ganzes betrachtest. Der Torus ist lokal flach, wie Ebene, Zylinder, Möbiusband und Kleinsche Flasche auch. Ein Musterbeispiel für Deine Aussage (sinngemäß), daß dieselbe lokale Geometrie des Raums unterschiedliche Topologien erlaubt.

Das würde ich nicht sagen. Eine verbeulte Sphäre ist topologisch immer noch eine Sphäre. So, wie eine Tasse topologisch immer noch ein Torus ist.
Ja, völlig richtig. Mit verbeult waren unterschiedliche lokale Krümmungen gemeint, was aber wahrscheinlich wenig Sinn macht.

Gruß, Timm

Bauhof
17.09.13, 17:09
Ich denke, hier ist es an der Zeit, die Genauigkeit der Messdaten ins Gespräch zu bringen. Man kann aus diesen (noch) nicht sagen, dass der Raum exakt flach ist. Nur, dass es ziemlich flach ist. Aber so, dass es immer noch auch leicht positive oder auch leicht negative Krümmung erlaubt.

Hallo Johann,

das habe ich auch schon seit längerer Zeit in mehreren Foren "gepredigt".

Wenn z.B. ein sphärisches Universum einen 4-D-Radius von 14 Milliarden Lichtjahren hat, dann ist es unmöglich mit den heutigen Messmethoden zu entscheiden, ob das Universum "exakt flach" oder doch eine kleine globale Krümmung aufweist.

Wenn es exakt flach ist, dann ist das Universum unendlich groß. Wenn es auch nur eine minimale gleichmäßige globale Krümmung hat, dann ist es zu jedem Zeitpunkt endlich groß. Und das ist letztendlich ein essentieller Unterschied.

M.f.G. Eugen Bauhof

Timm
17.09.13, 18:06
Hallo Eugen,

das habe ich auch schon seit längerer Zeit in mehreren Foren "gepredigt".
das würde ich an Deiner Stelle lieber nicht "predigen". Denn wenn man die heutige kleine Abweichung des Wertes von Omega von 1 als wahr annähme, dann wäre diese Abweichung zum Zeitpunkt des Urknalls gigantisch verkleinert, läge also Omega extrem nahe an 1. An einen solchen Zufall glauben die meisten Kosmologen nicht.

Falls das Universum räumlich flach ist, dann folgt übrigens keineswegs zwingend, daß es unendlich ist, siehe meine letzte Post an Johann.

Gruß, Timm

JoAx
17.09.13, 18:08
Laß Dich nicht täuschen ...


Das wäre ja der 'flache Torus', klar. Ich habe aber an einen "normalen" gedacht. Soll wohl heißen, dass so eins gar nicht erst betrachtet wird?


Ja, völlig richtig. Mit verbeult waren unterschiedliche lokale Krümmungen gemeint, was aber wahrscheinlich wenig Sinn macht.


Galaxien, Galaxienhaufen, Galaxiensuperhaufen sind ja schon solche lokalen "Beulen" und werden auch berücksichtigt ("ausgebügelt"). Eine lokale Krümmung, die wir aber nicht überblicken können, könnte schon "Probleme" bereiten. :D


Wenn z.B. ein sphärisches Universum einen 4-D-Radius von 14 Milliarden Lichtjahren hat, dann ist es unmöglich mit den heutigen Messmethoden zu entscheiden, ob das Universum "exakt flach" oder doch eine kleine globale Krümmung aufweist.

Wenn es exakt flach ist, dann ist das Universum unendlich groß. Wenn es auch nur eine minimale gleichmäßige globale Krümmung hat, dann ist es zu jedem Zeitpunkt endlich groß. Und das ist letztendlich ein essentieller Unterschied.


Fast, Eugen.

Dass du 14 Mrd. LJ als Radius anspricht, sagt mir, dass du das Alter des Universums mit dem Krümmungsradius (eine Art Hilfsgröße) verwechselst. Gerade bei R = 14 Mrd. LJ hätte man vermutlich locker erkant, dass die Geometrie nicht flach sein kann, sondern stark positiv ist.


Grüße, Johann

Timm
17.09.13, 18:59
Das wäre ja der 'flache Torus', klar. Ich habe aber an einen "normalen" gedacht. Soll wohl heißen, dass so eins gar nicht erst betrachtet wird?
Den Unterschied kenne ich nicht. Meinst Du den 2-Torus? Nein kaum, denn der ist lokal flach.

Gruss, Timm

amc
17.09.13, 20:46
Nur mal so in den "Raum" geworfen -

ich empfinde das irgendwie auffällig, aus meinem Blickwinkel des "gefährlichen (Physik-)Halbwissens", wenn überhaupt :) -

könnte es so sein, dass alle "Kräfte", und wohl voran die DE, so wirken, den "Raum" oder die "Raumzeit" global flach zu ziehen / erscheinen zu lassen?

Dann wäre die Expansion Folge von lokaler Krümmung?

Ich fände das irgendwie auch beruhigend, dann würden wir immer mehr oder weniger in vertrauter Euklidischer Geometrie leben. :)

Nur ein (spontaner) Gedanke, ohne tiefere Prüfung..

Grüße, amc

Jogi
17.09.13, 22:15
Ich fände das irgendwie auch beruhigend, dann würden wir immer mehr oder weniger in vertrauter Euklidischer Geometrie leben.

Da bin ich für meinen Teil bereits sehr beruhigt...:)

Bauhof
18.09.13, 09:22
Fast, Eugen. Dass du 14 Mrd. LJ als Radius anspricht, sagt mir, dass du das Alter des Universums mit dem Krümmungsradius (eine Art Hilfsgröße) verwechselst. Gerade bei R = 14 Mrd. LJ hätte man vermutlich locker erkant, dass die Geometrie nicht flach sein kann, sondern stark positiv ist.

Hallo Johann,

das verstehe ich jetzt nicht, dass bei R= 14 Mrd. LJ die Geometrie stark positiv sein soll.

Ich denke dabei an das Ballon-Modell, wobei die Ballonoberfläche unseren Raum (oder unsere Raumzeit) symbolisiert. Wenn die innere Krümmung der Ballonoberfläche 1/14 Mrd. LJ beträgt, dann ist doch m.E. die Geometrie der Ballonoberfläche nicht stark positiv, sondern schwach positiv und kaum nachweisbar.

M.f.G. Eugen Bauhof

JoAx
18.09.13, 17:22
Ich denke dabei an das Ballon-Modell, wobei die Ballonoberfläche unseren Raum (oder unsere Raumzeit) symbolisiert. Wenn die innere Krümmung der Ballonoberfläche 1/14 Mrd. LJ beträgt, dann ist doch m.E. die Geometrie der Ballonoberfläche nicht stark positiv, sondern schwach positiv und kaum nachweisbar.


Ok. Kannst du das bitte zeichnen.

Eyk van Bommel
19.09.13, 07:21
Kommt es mir nur so vor oder gibt es hier (in diesem Themenbereich) nur gemeinsame Schnittstellen und wenige (großflächige) Überlagerungen bei Thema Topologie und innere/äußere Krümmung.
Und wenn ich dann noch den Satz von „ICH“ im Nachbarforum lese (http://www.quantenforum.de/viewtopic.php?p=2317&sid=d94cf7214a3e7c74756e6d1f44196d59#p2317):
ZITAT: ICH
Was für ein Schwachsinn. Mal doch mal ein Dreieck auf deine henkellose Kaffeetasse (Winkelsumme: 180°) und form sie dann um in eine Kugel (Winkelsumme >180°). Innere Krümmungen verändern sich höchst offensichtlich generell, wenn sich die äußere Krümmung ändert.

Dann bin ich endgültig verwirrt.

Leider habe ich das Thema in einer Zeit angefangen in der ich aus beruflichen und privaten Gründen gerade wenig teilnehmen kann.
Aber ich möchte doch eine Frage in den Raum werfen. Eine nach dem KP überall vorzufindende positive Krümmung würde doch eine Hypersphäre ohne die Notwenigkeit einer äußeren Krümmung bewirken :confused:

Und ich glaube das ist das worauf Eugen hinaus will?

Gruß
EVB

JoAx
19.09.13, 08:42
Alex, ich weiss, dass wir (die anderen) da wieder etwas "zur Seite" abgewichen sind, aber das heisst ja nicht, dass du uns auch folgen musst. Bleib dort, wo du mitkommst. Hasst du über mein letzten Post an dich nachgedacht?


Eine nach dem KP überall vorzufindende positive Krümmung würde doch eine Hypersphäre ohne die Notwenigkeit einer äußeren Krümmung bewirken :confused:


Die riemannschen Mannigfaltigkeiten brauchen nicht eingebettet zu sein. Und weil das so ist, weil eine zusätzliche Dimension nur eine Hilfsdimension ohne physikalische Bedeutung wäre, betrachtet man die Sachen auch ohne dieser. Ansonsten (für den ganz persönlichen Anfang) kann man auch mit so einer Hilfsdimension arbeiten. Einstein hat es bsw. genau so getan.

Wenn wir ganz, ganz, ganz ehrlich sind, dann können wir die Frage, ob unsere "Welt" eingebettet ist oder nicht - prinzipiell nicht beantworten. Die Physik braucht die Antwort auf diese Frage (oder die Frage selbst) aber auch nicht.


Und ich glaube das ist das worauf Eugen hinaus will?


Mal schauen.


Grüße

Bauhof
19.09.13, 13:50
Zitat von Bauhof
Ich denke dabei an das Ballon-Modell, wobei die Ballonoberfläche unseren Raum (oder unsere Raumzeit) symbolisiert. Wenn die innere Krümmung der Ballonoberfläche 1/14 Mrd. LJ beträgt, dann ist doch m.E. die Geometrie der Ballonoberfläche nicht stark positiv, sondern schwach positiv und kaum nachweisbar.

Ok. Kannst du das bitte zeichnen.

Anhand der von John D. Barrow erwähnten "Krümmungsuhr" habe ich eine Zeichnung gemacht, siehe Anhang. John D. Barrow schreibt auf Seite 238 seines Buches [1] folgendes:

Früher oder später aber wird es sogar für Atome und Nukleonen – und für überhaupt alles aus Materie – zu heiß. Was aber verstehen wir unter Zeit, wenn nichts mehr übriggeblieben ist, um sie zu messen?

Nun, vielleicht gibt es eine universale Uhr, die nicht die Eigenzeit misst. Eine solche Uhr müsste mit etwas verbunden sein, was immer da ist, auch zum Zeitpunkt der Singularität. Der einzige Kandidat, der dafür in Frage käme, dürfte wohl die Krümmung des Raums beziehungsweise die Materiedichte sein. Eine solche Uhr könnte jedermann im Weltall benutzen. In dem Augenblick aber, in dem wir die Krümmungsuhr zum Chronometer der wirklichen Zeit erklären, müssen wir einige recht ungewöhnliche Konsequenzen gewärtigen.

Barrow geht offenbar davon aus, dass das Universum zu Beginn stark gekrümmt war, weil der Krümmungsradius noch sehr klein war. Zur heutigen Zeit ist R(t) auf etwa 14 Milliarden Lichtjahre angewachsen, die Krümmung würde 1/R(t) betragen, also ein sehr kleiner Wert und damit nur noch eine schwache positive Krümmung, die mit Triangulation nicht nachweisbar wäre. Carl Friedrich Gauß versuchte mal eine mögliche Raumkrümmung auf der Erde durch Triangulation nachzuweisen und ist daran gescheitert. Auf dem alten 10-DM-Schein war dieser Versuch abgebildet

Nur im frühen Universum könnte eine Raumkrümmung nachweisbar sein, weil hier die mögliche Raumkrümmung noch viel größer war. Siehe dazu auch den Beitrag
Ist das Universum leicht gekrümmt?. (http://www.quanten.de/forum/showthread.php5?t=2477)

M.f.G. Eugen Bauhof

P.S.
R(t) wird in der kosmologischen Literatur als "Maßfaktor" bezeichnet. Beim Ballonmodell ist es der Radius. Der Begriff Maßfaktor wird aber auch in anderen Universum-Modellen verwendet.

[1] Barrow, John D. und Silk, Joseph
Die linke Hand der Schöpfung.
Der Ursprung des Universums.
Heidelberg 1995
ISBN=3-86025-355-7

JoAx
19.09.13, 16:15
Eugen, das geht an meiner Frage "ein wenig" vorbei.

Du meinst ungefähr schlicht - größerer Kreis => kleinere Krümmung. Das ist aber etwas naiv, wenn man so sagen darf. Was ist Krümmung? Das ist ein Mass dafür, wie stark ein Stück einer Linie von der Geraden abweicht. Ein "voller Kreis" weicht in jeder Größe gleich stark von einer Geraden ab. 1/4 eines Kreises weicht aber weniger von der Geraden ab, als bsw. 1/2 des selben Kreises.

Heute können wir, wenn wir CMB betrachten, bis ca. 380 Tausend Jahre nach dem Urknall sehen. Das heisst - fast genau so weit, wie das Universum Alt ist.

...

Verstehst du, worauf ich hinaus möchte?

Bauhof
19.09.13, 17:42
Verstehst du, worauf ich hinaus möchte?

Hallo Johann,

leider nicht.

M.f.G. Eugen Bauhof

JoAx
19.09.13, 18:25
leider nicht.


Nimm vom Umfang eines Kreises genau so viel, wie sein Radius ist. Dann wird die Gesamtkrümmung dieses Stücks immer gleich sein, egal, wie groß du den Kreis wählst. Die Krümmung, die du mit K=1/R meinst, ist eine lokale Krümmung, Krümmung an einem "Punkt". Diese könnte (in deinem Sinne) nur die Flachheit des Raumes in unserer unmittelbaren Umgebung erklären, nicht aber die auf grossen Skalen = bis hin zum CMB.

Krümmung einer Fläche (http://de.wikipedia.org/wiki/Kr%C3%BCmmung#Kr.C3.BCmmung_einer_Fl.C3.A4che)

und das nächste Kapitel.

Grüße

Eyk van Bommel
19.09.13, 18:34
Ich denke JoAx geht davon aus, dass wenn wir bis zum Anfang sehen könnten, dann sehen wir den ganzen Kreis. Der ganze Kreis weicht natürlich mehr von einer Geraden ab.

Eugen geht aber imho davon aus, dass das Universum das wir sehen auch wenn wir den Anfang betrachten nur ein Bruchteil der Sphäre ausmacht.

Somit müsste das Universum bereits 400 TJ nach dem Urknall bereits praktisch flach gewesen sein.


Gruß
EvB

JoAx
19.09.13, 18:40
Eugen geht aber imho davon aus, dass das Universum das wir sehen auch wenn wir den Anfang betrachten nur ein Bruchteil der Sphäre ausmacht.

Somit müsste das Universum bereits 400 TJ nach dem Urknall bereits praktisch flach gewesen sein.


Ich gehe auch davon aus, dass das Universum zu der angesprochenen Zeit ziemlich flach gewesen ist. Das ist schon die korrekte Richtung, Alex, aber jetzt muss man weiter denken .... ;)

Bauhof
20.09.13, 09:39
Nimm vom Umfang eines Kreises genau so viel, wie sein Radius ist. Dann wird die Gesamtkrümmung dieses Stücks immer gleich sein, egal, wie groß du den Kreis wählst. Die Krümmung, die du mit K=1/R meinst, ist eine lokale Krümmung, Krümmung an einem "Punkt". Diese könnte (in deinem Sinne) nur die Flachheit des Raumes in unserer unmittelbaren Umgebung erklären, nicht aber die auf grossen Skalen = bis hin zum CMB.

Krümmung einer Fläche (http://de.wikipedia.org/wiki/Kr%C3%BCmmung#Kr.C3.BCmmung_einer_Fl.C3.A4che)

und das nächste Kapitel.

Hallo Johann,

ja, die Krümmung, die ich mit K=1/R [1] meinte, ist eine Krümmung in einem Punkt des Kreises. Ich verstehe nicht den Unterschied zwischen lokaler Krümmung und Gesamtkrümmung. Insbesondere verstehe ich nicht, wie du die Gesamtkrümmung eines Stücks des Kreises definierst.

Wie definierst du die Gesamtkrümmung eines Stücks des Kreises? Wie die Gesamtkrümmung eines Teils der Kugeloberfläche? Wie die Gesamtkrümmung eines Teils des dreidimensionalen Raumes?

M.f.G. Eugen Bauhof

[1] Nach Gauß (http://de.wikipedia.org/wiki/Gau%C3%9Fsche_Kr%C3%BCmmung) muss es für eine Kugeloberfläche K=1/R² heißen und für eine dreidimensionale Sphäre vermutlich K=1/R³.

Ich
20.09.13, 13:13
das verstehe ich jetzt nicht, dass bei R= 14 Mrd. LJ die Geometrie stark positiv sein soll.

Ich denke dabei an das Ballon-Modell, wobei die Ballonoberfläche unseren Raum (oder unsere Raumzeit) symbolisiert. Wenn die innere Krümmung der Ballonoberfläche 1/14 Mrd. LJ beträgt, dann ist doch m.E. die Geometrie der Ballonoberfläche nicht stark positiv, sondern schwach positiv und kaum nachweisbar.

Die Abweichung von der kritischen Dichte, die einem flachen Raum entspricht, ist nach den neuesten Daten (Hier (http://arxiv.org/pdf/1303.5076v1.pdf), S. 40, leider wieder nur auf Englisch) mit einer Sicherheit von 95% kleiner als 0,7%. Diese Zahl heißt \Omega_k.
Für den Krümmungsradius c/(H*sqrt(\Omega_k)) ergibt sich daraus, wenn wir c/H auf 14 Mrd Lichtjahre setzen, ein Mindestwert von ca. 170 Mrd Lj.

Auf derselben Seite werden übrigens auch potentielle "lokale" Krümmungsfluktuationen (sprich: von der Größenordnung des beobachtbaren Universums) erwähnt sowie die mögliche leichte negative Krümmung, die sich in Modellen zeigt, wie sie im anderen Thread (http://www.quanten.de/forum/showthread.php5?t=2477) besprochen werden.

Bauhof
20.09.13, 13:49
Die Abweichung von der kritischen Dichte, die einem flachen Raum entspricht, ist nach den neuesten Daten (Hier (http://arxiv.org/pdf/1303.5076v1.pdf), S. 40, leider wieder nur auf Englisch) mit einer Sicherheit von 95% kleiner als 0,7%. Diese Zahl heißt \Omega_k.
Für den Krümmungsradius c/(H*sqrt(\Omega_k)) ergibt sich daraus, wenn wir c/H auf 14 Mrd Lichtjahre setzen, ein Mindestwert von ca. 170 Mrd Lj.

Hallo ICH,

heißt das, dass man aufgrund dieser Abweichung von 0,7% vom flachen Raum ein Ballonmodell mit einem Mindestradius von 170 Milliarden Lichtjahren nicht ausschließen kann? So das der Weltraum z.B. eine 3-D-Sphäre mit diesem Mindest-Radius sein könnte?

M.f.G. Eugen Bauhof

Ich
21.09.13, 22:24
Genau. 14 Mrd wäre deutlich zu stark gekrümmt, 170 Mrd könnte aber sein.
Beobachtungen können prinzipiell nur einen Mindestradius ergeben, die Frage nach exakter Flachheit oder nicht ließe sich nur theoretisch klären, wenn sich z.B. irgendwelche überzeugenden Gründe nennen ließen, warum es eigentlich nur so oder so sein kann. Dem ist heutzutage nicht so: die Inflation zieht alles glatt, behält aber im Prinzip die Krümmung des ursprünglichen Raumbereichs bei. Der mag flach und wegen mir sogar ein Torus gewesen sein, oder auch sphärisch oder hyperbolisch. Es gibt einfach noch keine Theorie, die dazu irgendwas Belastbares sagen würde.

Bauhof
22.09.13, 09:47
Genau. 14 Mrd wäre deutlich zu stark gekrümmt, 170 Mrd könnte aber sein.
Beobachtungen können prinzipiell nur einen Mindestradius ergeben, die Frage nach exakter Flachheit oder nicht ließe sich nur theoretisch klären, wenn sich z.B. irgendwelche überzeugenden Gründe nennen ließen, warum es eigentlich nur so oder so sein kann. Dem ist heutzutage nicht so: die Inflation zieht alles glatt, behält aber im Prinzip die Krümmung des ursprünglichen Raumbereichs bei. Der mag flach und wegen mir sogar ein Torus gewesen sein, oder auch sphärisch oder hyperbolisch. Es gibt einfach noch keine Theorie, die dazu irgendwas Belastbares sagen würde.

Hallo ICH,

so sehe ich es auch, dass es dazu noch keine Theorie gibt, die irgendwas Belastbares sagen würde. Deshalb wundert es mich, dass man überall liest, dass das Universum flach sei und es außer Betracht läßt, dass es sattdessen ganz schwach in sich selbst zurückgekrümmt sein könnte, so das der Rauminhalt des Universums auch endlich sein könnte.

Max Planck hat erkannt, dass es keine unendlich kleine Energie-Portionen geben kann. David Hilbert postulierte, dass das Unendliche nirgends realisiert ist. Aber ein unendlich großes Universum will man zur Zeit noch nicht ausschließen.

M.f.G. Eugen Bauhof

Marco Polo
22.09.13, 12:05
die Inflation zieht alles glatt, behält aber im Prinzip die Krümmung des ursprünglichen Raumbereichs bei. Der mag flach und wegen mir sogar ein Torus gewesen sein, oder auch sphärisch oder hyperbolisch. Es gibt einfach noch keine Theorie, die dazu irgendwas Belastbares sagen würde.

Im Grunde erscheint der Raum nach der Inflation immer flach, egal wie die Ausgangssituation war. Unser Beobachtungshorizont ist so klein, dass eben alles flach erscheint, auch wenn dies auf grösseren Skalen vielleicht gar nicht der Fall ist.

Die beschleunigte Expansion (Dunkle Energie) führt die Inflation quasi fort, wenn auch moderater.

Interessant finde ich in diesem Zusammenhang übrigens auch die Tatsache, dass bei einem dynamischen Universum der Enegieerhaltungssatz nicht mehr gilt. Bei näherer Betrachtung ist das aber völlig logisch, finde ich. Ist aber eh ein anderes Thema.

Grüsse, MP

Marco Polo
22.09.13, 12:15
Aktuell ist das Forum quälend langsam. Mausklicks werden bei mir teilweise erst nach Minuten quittiert. Teilweise gehen auch Formatierungen verloren, die ich nachträglich einpflegen muss.

Andere Seiten im Web reagieren hingegen blitzschnell.

Geht euch das auch so?

Bauhof
22.09.13, 12:35
Aktuell ist das Forum quälend langsam. Mausklicks werden bei mir teilweise erst nach Minuten quittiert. Teilweise gehen auch Formatierungen verloren, die ich nachträglich einpflegen muss.

Andere Seiten im Web reagieren hingegen blitzschnell.

Geht euch das auch so?

Hallo Marc,

ja, das beobachte ich schon seit etwa einer Woche. Ich benutze Windows 7 und den Internet-Explorer 10. Wir sollten es Günter mitteilen, denn ein normaler Forenbetrieb ist bald nicht mehr möglich, weil die Antwortzeiten immer länger werden.

M.f.G. Eugen Bauhof

Marco Polo
22.09.13, 12:46
Würdest du das übernehmen, Eugen? Muss jetzt los. :)

Bauhof
23.09.13, 10:01
Würdest du das übernehmen, Eugen? Muss jetzt los. :)

Hallo Marc,

ich habe es Günter per E-Mail gemeldet.
Er teile mir mit, dass das Problem innerhalb der nächsten 48 Stunden von seinem Provider behoben wird.

M.f.G. Eugen Bauhof

Marco Polo
23.09.13, 17:56
Supi. Danke Eugen. :)

Eyk van Bommel
24.09.13, 09:51
Nachdem die "Forenproblematik" in Arbeit ist – kann man es wegen der Übersichtlichkeit wieder entfernen/verschieben?

Zum Thema: Es gibt nun zwei Aussagen die ich derzeit einfach nicht wirklich sinnvoll zusammen bringen kann.

A) Der Winkel in einem Dreieck ändert sich nicht, wenn man eine äußere Krümmung einfügt. (Papier und Zylinder)

B) Wenn man ein Dreieck in die "Delle einer Hypersphäre" zeichnet und man diese durch "aufblasen" dann wieder entfernt, dann ändert sich der Winkel.

Hängt es nun von der Art der äußeren Krümmung ab, ob sie Einfluss auf die Innere hat? Oder stellt die Delle eine innere Krümmung in einer Hypersphäre dar?

Gruß
Axel

Eyk van Bommel
26.09.13, 22:54
Hab ich das Thema verfehlt?

Ich
27.09.13, 08:21
Hier (http://books.google.de/books?id=OeSdXNimRKUC&pg=PA311&lpg=PA311&dq=innere+und+%C3%A4u%C3%9Fere+kr%C3%BCmmung&source=bl&ots=4PS7jmzeHf&sig=SOHuKh40mrSEOXcJPNoHAFUFEdI&hl=de&sa=X&ei=6C5FUp8Sw9-zBr_tgdgM&ved=0CDQQ6AEwATgK#v=onepage&q&f=false)ist eine Definition der Begriffe.

Wenn man eine Fläche in einen geeigneten höheren (flachen) Raum einbettet, dann wird die innere Krümmung der Fläche immer als äußere Krümmung sichtbar. Zum Beispiel als Kugel. Wenn man die äußere Krümmung der Kugel verändert, indem man eine Delle hineindrückt, dann ändert sich dadurch auch die innere Krümmung der Fläche.
Nur in bestimmten Fällen kann man die äußere Krümmung ändern, ohne die innere Krümmung anzutasten.

Deswegen hatte ich seinerzeit geschrieben:
Innere Krümmungen verändern sich höchst offensichtlich generell, wenn sich die äußere Krümmung ändert.
Generell ändert sie sich, in Spezialfällen nicht.
Konfusion kann aufkommen, wenn einer mit "äußerer Krümmung" ausschließlich Krümmungen meint, die sich im Inneren nicht feststellen lassen, ein anderer aber nicht.

Wobe "äußere Krümmung" für die ART irrelevant ist.

Bauhof
27.09.13, 16:47
Hier (http://books.google.de/books?id=OeSdXNimRKUC&pg=PA311&lpg=PA311&dq=innere+und+%C3%A4u%C3%9Fere+kr%C3%BCmmung&source=bl&ots=4PS7jmzeHf&sig=SOHuKh40mrSEOXcJPNoHAFUFEdI&hl=de&sa=X&ei=6C5FUp8Sw9-zBr_tgdgM&ved=0CDQQ6AEwATgK#v=onepage&q&f=false)ist eine Definition der Begriffe.

Wenn man eine Fläche in einen geeigneten höheren (flachen) Raum einbettet, dann wird die innere Krümmung der Fläche immer als äußere Krümmung sichtbar. Zum Beispiel als Kugel. Wenn man die äußere Krümmung der Kugel verändert, indem man eine Delle hineindrückt, dann ändert sich dadurch auch die innere Krümmung der Fläche.
Nur in bestimmten Fällen kann man die äußere Krümmung ändern, ohne die innere Krümmung anzutasten.

Deswegen hatte ich seinerzeit geschrieben:

Generell ändert sie sich, in Spezialfällen nicht.
Konfusion kann aufkommen, wenn einer mit "äußerer Krümmung" ausschließlich Krümmungen meint, die sich im Inneren nicht feststellen lassen, ein anderer aber nicht.

Wobe "äußere Krümmung" für die ART irrelevant ist.

Hallo ICH,

ich habe mir einen Textabschnitt aus der Leseprobe (http://books.google.de/books?id=OeSdXNimRKUC&pg=PA311&lpg=PA311&dq=innere+und+%C3%A4u%C3%9Fere+kr%C3%BCmmung&source=bl&ots=4PS7jmzeHf&sig=SOHuKh40mrSEOXcJPNoHAFUFEdI&hl=de&sa=X&ei=6C5FUp8Sw9-zBr_tgdgM&ved=0CDQQ6AEwATgK#v=onepage&q&f=false) kopiert:

312 7. Axiome, Definitionen und Postulate der Geometrie.

So ist z.B. jeder Punkt der Erdoberfläche durch zwei Zahlen (geographische Breite und Länge) bestimmbar. Wenn wir uns nun für die Stärke der Krümmung der Fläche interessieren, so gehen wir gewöhnlich aus der zweidimensionalen Fläche in den dreidimensionalen Raum über:

Wir können z.B. die Stärke der Krümmung durch Angabe der Lage des Mittelpunktes der Kugel bestimmen, oder allgemein durch Angabe von Krümmungsmittelpunkten. Diese liegen aber nie in der gekrümmten Fläche selbst, sie sind daher nur dreidimensional zu bestimmen. Man kann die so bestimmte Krümmung einer Fläche auch als äußere Krümmung bezeichnen, weil es zu ihrer Feststellung notwendig ist, das zweidimensionale Gebilde selbst zu verlassen.

Gauss hat nun gezeigt, daß es möglich ist, die Krümmung einer Fläche zu bestimmen, ein Maß der Krümmung zu finden, ohne aus dem zweidimensionalen Bereich der Fläche selbst hinauszugehen. Man kann dann die so festgestellte Krümmung auch als die innere Krümmung der Fläche bezeichnen.

Wenn ich das richtig verstehe, dann heißt das, dass sich die Begriffe innere und äußere Krümmung nur dadurch unterscheiden, mit welcher Methode sie bestimmt wurden. Einmal wird die Krümmung einer Kugeloberfläche unter Zuhilfenahme der dritten Dimension über den Kugelradius bestimmt. Nach Gauß kann man die Krümmung derselben Kugeloberfläche bestimmen, ohne die dritte Dimension zur Hilfe zu nehmen. Da gibt es keinen quantitativen Unterschied zwischen innerer und äußerer Krümmung.

M.f.G. Eugen Bauhof

Ich
27.09.13, 21:06
Hallo Eugen,

ja, ich denke, so kann man das sagen. Die innere Krümmung ist in unserem Zusammenhang besonders wertvoll, weil sie in der Raumzeit selbst definiert ist und keine unbeobachtbaren Zusatzdimensionen braucht. Der Vorstellung zugänglicher wird sie aber, wenn man sie sich als äußere Krümmung von Flächen vorstellt.

Eyk van Bommel
04.10.13, 15:52
Langsam befürchte ich, dass das Ballon-Modell besser zu meinem Kopf passt wie zum Universum :D

Nun was habe ich gelernt?

Die äußere Krümmung des Universums ist/wäre nur eine andere mathematische Darstellung der inneren Krümmung – wenn denn vorhanden.

Eine äußere Krümmung ist nur dann nicht durch eine innere Krümmung eindeutig darstellbar, wenn sie sowieso keinen Einfluss auf die innere hat? Gerader Zylinder/Kegel??
Aus: http://quanten.de/forum/showpost.php5?p=73456&postcount=85
Nach Gauß kann man die Krümmung derselben Kugeloberfläche bestimmen, ohne die dritte Dimension zur Hilfe zu nehmen.
Aus:http://quanten.de/forum/showpost.php5?p=73458&postcount=86
Die innere Krümmung ist in unserem Zusammenhang besonders wertvoll, weil sie in der Raumzeit selbst definiert ist und keine unbeobachtbaren Zusatzdimensionen braucht.
Nun "schon" Gauß braucht keine äußere Krümmung. Wenn Gauß keine höhere Dimension benötigt wieso solle es in der RT anderes sein? Oder ist der Satz tiefgründiger zu verstehen.

Alles in allem bleibt bei mir das Problem bestehen, wie man die Büchse der Pandora schließen soll, wenn sich der Deckel mit c entfernt.:confused:

Oder anders: Das „G-Feld“ der Energie eines SL kollabiert auch nicht "mit ihm". Auch nicht das Feld das im Moment der Entstehung bereits innerhalb liegt. Für mich sollte die Raumkrümmung erhalten bleiben auch nach dem "Big Crunch".

Es ist auch nicht so, dass ich das Bild nicht verstehe: Dass für jeden Beobachter an jedem Punkt dasselbe Bild entsteht – genauso wie es sich auf einer Hypersphäre / Ballon eben darstellt - aber gilt dies auch für masselose Teilchen?

Es ist vielmehr so, dass doch die Frage nach der „potentiellen Energie“ und der „Fluchtgeschwindigkeit“ in einem tatsächlich randlosem isotropen Universum keinen Sinn macht (???)! Oder?
Und gerade die potentiellen Energie“ vs. „Fluchtgeschwindigkeit“ steckt doch (irgendwie) in der Friedman-Gleichung – oder ist dies nur die didaktisch reduzierte Wahrheit.

Das globale G-Feld einer Hypersphäre ist doch an jedem Ort "Null" - so wie im inneren ein "Kugel" oder eines " "Ringes" auf einem Gummituch. "

:confused: :confused:

Gruß
EvB

Ich
05.10.13, 12:03
Es ist vielmehr so, dass doch die Frage nach der „potentiellen Energie“ und der „Fluchtgeschwindigkeit“ in einem tatsächlich randlosem isotropen Universum keinen Sinn macht (???)! Oder?
Ja.
Und gerade die potentiellen Energie“ vs. „Fluchtgeschwindigkeit“ steckt doch (irgendwie) in der Friedman-Gleichung
Ja.
Wenn du dir eine kleine Kugel aus dem Universum ausgeschnitten denkst und in einen sonst leeren Raum versetzt, dann haben die Begriffen einen Sinn. Und wie bereits erläutert, wird sich das ganze Universum, egal welche Topologie es hat, genauso verhalten wie die Kugel. Darum haben die Begriffe im Friedmann-Modell ihre Bedeutung.
Das globale G-Feld einer Hypersphäre ist doch an jedem Ort "Null" - so wie im inneren ein "Kugel" oder eines " "Ringes" auf einem Gummituch. "
Was Ähnliches hatten wir hier schon mal disuktiert, hier (http://www.quanten.de/forum/showthread.php5?t=2369).

Eyk van Bommel
05.10.13, 19:43
Hallo ICH,
deine Antworten hier und im JoAx-Thread versuche ich noch zu verarbeiten.

Hab aber noch'ne triviale Frage. Benötigt eine Krümmung in einem räumlich offenen Universum (z.B Schale) nicht immer eine Art Unsymmetrie? Ich meine damit, eine solche Krümmung benötigt eine Richtung:rolleyes: . Nach oben oder unten? Links oder rechts?

Gruß
EvB

Ich
05.10.13, 20:51
Nein, keine Richtung. Auf der Kugel gibt's auch keine ausgezeichneten Richtungen.

Eyk van Bommel
11.10.13, 15:31
Hallo Ich,

danke für deine Antworten. Offenbar komme ich aber damit nicht zurecht.

Da ich nicht daran zweifle, dass deine Aussagen richtig sind muss ich einfach Kapitulieren. Oder du/ein anderer hast Lust mir weiter zu helfen, aber man muss dann wohl in Kauf nehmen, dass ich mich noch eine Weile im Kreis drehe.

Für mich macht es einen Unterschied, ob ein Punkt im/nach Beginn seiner Ausdehnung bereits eine Kugel oder eine Schale ist.

Ich stimme dir zu, dass in diesem Fall (Punkt->Kugel) die Krümmung keine bevorzugte Richtung aufweist, aber bei einer Schale ist das imho nicht so.

Zumindest, wenn man sie eingebettet in einer höheren Dimension sieht.
Wenn du dir eine kleine Kugel aus dem Universum ausgeschnitten denkst und in einen sonst leeren Raum versetzt, dann haben die Begriffe einen Sinn.
Ja dann schon – aber das ist kein (in sich) geschlossener Raum - kein Rand?

In deinem Link habe ich leider keine ausreichende Hilfe für meine Fragestellung gefunden.

Ein „kosmologischer Staubtorus“ ist keine Hypersphäre in dem auch das Gravitationsfeld „geschlossen“ ist? Das globale G-Feld kann in einer perfekten Sphäre keine Richtung aufweisen und somit existiert es nicht.

Während die Teilchen im „Staubtorus“sich bewegen müssen, damit die Teilchen nicht zusammenstürzen ("Big Crunch") müssen es die Teilchen in einer perfekten Sphäre/Torus nicht.

--------------------------------------------------------------------
Zu Pandora, Wkin/Wpot und dem „Big Crunch“.

Ich bin zu dem Schluss gekommen, dass der Deckel/das Photon/das masselose Teilchen bei einem „Big Crunch“ nicht räumlich „mit kollabiert“ müssen - sondern eigentlich während der Flucht vor dem „Crunch“ immer mehr ins rote verschoben werden (=Der Abstand wird kleiner das Photon „röter“). Und wenn alle Masse zusammengefallen ist, dann ist auch das letzte Photon durch diesen Prozess verschwunden.

Zumindest komme auch ich dann zu einem vollständigen Kollaps. :)

Gruß
EvB

Ich
14.10.13, 15:42
Ich stimme dir zu, dass in diesem Fall (Punkt->Kugel) die Krümmung keine bevorzugte Richtung aufweist, aber bei einer Schale ist das imho nicht so.

Zumindest, wenn man sie eingebettet in einer höheren Dimension sieht.
Jetzt mal abgesehen davon, dass eine exakte Einbettung einer homogenen negativ gekrümmten Fläche in drei Dimensionen tatsächlich "schwierig" ist, so sehe ich doch nicht, wo da die ausgezeichnete Richtung sein soll. Und wieso du das Gebilde überhaupt "Schale" nennst, es ist eine Sattelfläche.
Wohlgemerkt dient die Einbettung nur der Illustration, eine ausgezeichnete Richtung muss sich schon innerhalb der Fläche nachweisen lassen. Alles andere sind nur Artefakte, die sich Leute einhandeln, die die Einbettung als wichtiger ansehen als die innere Geometrie der Fläche.
Ja dann schon – aber das ist kein (in sich) geschlossener Raum - kein Rand?
Der hätte sogar einen Rand, die Kugeloberfläche nämlich, im Gegensatz zu allen Universumsmodellen.
Is ja egal, auf jeden Fall funktionieren dort die ganzen Newtonschen Begriffe, weil man eben nicht mit nichttrivialen Topologien rumspielt. Man kann also die Dynamik dieser Kugel nach Newton berechnen.
Interessant wird es dadurch, dass alle Universen, egal wie sie aussehen, genau derselben Dynamik folgen müssen und deswegen die Newtonschen Gleichungen eine gewisse Bedeutung in der Kosmologie haben.

Dieter van Bohr
14.10.13, 20:08
Ich denke das sich alles wieder zusammenzieht, da irgendwann alle Energie aufgebraucht ist und das einzige was bleibt ist die Gravitation und letztendlich entsteht dann ein riesiger Planet, oder am ende gibt es zwei riesen Planeten die sich dann gegenseitig anziehen und letztendlich mit enormer Geschwindigkeit aufeinandertreffen und damit eventuell zu einem neuen Urknall führen. Deswegen denke ich das es niemals enden wird, sondern immerwieder neues erschafft.

Eyk van Bommel
14.10.13, 20:23
Hallo ICH,
Der hätte sogar einen Rand, die Kugeloberfläche nämlich, im Gegensatz zu allen Universumsmodellen.
Ich hatte mich dumm ausgedrückt. Kein Rand bezog sich auf meine eigene Aussage. Daher ja - hier hat das Universum einen Rand. Dieses Universum wäre räumlich endlich. Hier passt Newton... Und auch Friedmann...

Bei einer Hypersphäre gibt es nur einen zur nächst höheren Dimension. Da die räumliche Ausdehnung hier null ist - spielt sie keine Rolle. Wenn ich das so sagen darf.

Aber nur in einem räumlich geschlossenen Universum (z.B Hypershäre) wäre es denkbar sich selbst zu erschießen in dem man die Mündung der Waffe von sich weg hält.
Und wieso du das Gebilde überhaupt "Schale" nennst, es ist eine Sattelfläche...
Der Begriff "Schale" resultiert aus dem Thread von Bauhof.
Ist das Universum leicht gekrümmt? (http:// http://quanten.de/forum/showpost.php5?p=73406&postcount=1) siehe Link.

Ich ging davon aus, dass sie Sattelfläche schreiben, wenn sie diese meinen. Sattelfläche wäre negativ gekrümmt - Eine Schale wäre (für mich) wie ein kreisrundes Stück aus einer Hypersphäre (Kugelausschnitt). Räumlich offen aber positive Krümung.

Ein "Stück/Ausschnitt" Sattelfläche vs Torus wäre aber dasselbe in grün.

Alles andere sind nur Artefakte, die sich Leute einhandeln, die die Einbettung als wichtiger ansehen als die innere Geometrie der Fläche.
Das stimmt schon. Ich hatte mich nur gefragt - warum "so herum". Schale nach "oben" oder nach "unten" gekrümmt. Bei einem räumlich geschlossenen System stellt sich die Frage einfach nicht. Das gilt natürlich nur aus Sicht einer höheren Dimension. Das ist aber nicht wichtig/richtig. Schale nach oben/unten - sie hätte dieselbe innere Krümmung. Also egal.

--------------------------------------------------------------
Du schreibst - man benötigt keine höhere Dimension. O.K. Aber muss das Modell nicht auch noch in sich stimmig sein wenn man eine solche annimmt?

Gauß hat gezeigt, dass die innere Krümmung in der Geometrie reicht - aber da ist keine Dynamik inkludiert. Kann man die Dynamik eines Luftballons und deren Kräfte (Vektoren) ohne 3. Dimension verstehen?

Gruss
EvB

Ich
15.10.13, 09:14
Du schreibst - man benötigt keine höhere Dimension. O.K. Aber muss das Modell nicht auch noch in sich stimmig sein wenn man eine solche annimmt?
Nein. Es ist dein Privatvergnügen, Räume in beliebige Anzahlen weiterer Dimensionen einzubetten. Das mag funktionieren oder nicht (bei ausreichend vielen Dimensionen funktioniert es immer), hat aber nichts mit Gravitationsphysik zu tun.
Gauß hat gezeigt, dass die innere Krümmung in der Geometrie reicht - aber da ist keine Dynamik inkludiert. Kann man die Dynamik eines Luftballons und deren Kräfte (Vektoren) ohne 3. Dimension verstehen?
Man kann die Dynamik eines Luftballons nicht ohne 3. Dimension verstehen. Aber man kann die Dynamik des Universums ohne zusätzliche 4. Dimension verstehen.

Wie schon mehrfach erwähnt: Die ART ist ganz einfach. Wenn ein kleiner Ausschnitt des Universums, in leeren Raum verbracht, diese oder jene Dynamik hätte, dann ist das auch die Dynamik des ganzen Universums. Fertig.

Timm
16.10.13, 14:43
Im Grunde erscheint der Raum nach der Inflation immer flach, egal wie die Ausgangssituation war. Unser Beobachtungshorizont ist so klein, dass eben alles flach erscheint, auch wenn dies auf grösseren Skalen vielleicht gar nicht der Fall ist.

Die beschleunigte Expansion (Dunkle Energie) führt die Inflation quasi fort, wenn auch moderater.


Das sehe ich ein bißchen anders. Bei einem Raum konstanter Krümmung findet man lokal überall dieselbe Krümmung. Vielleicht spielst Du auf die Topologie an. Wenn man sich den Torus als Ganzes vorstellt, erscheint er zwar gekrümmt (das meinst Du vielleicht mit "größere Skalen"), ist aber dennoch an jeder Stelle flach.

Zu Deinem ersten Satz volle Zustimmung. Bemerkenswert finde ich in diesem Zusammenhang die Unterscheidung beschleunigte, bzw. verlangsamte vs. exponentielle Expansion. Nach Friedmann gilt

((1-Ω)/Ω)ρa² = const.

Nach dem Standardmodell (ohne Inflation) nimmt a² langsamer zu, als die Dichte ρ abnimmt. Berücksichtigt man das, nimmt seit der Planck Ära ρa² um den Faktor ~ 10^30 ab und somit (1-Ω)/Ω) entsprechend zu. Als Resultat divergieren anfängliche Abweichungen von Ω = 1 (-> Dichte = kritische Dichte) rasant.
Umgekehrt Mit Inflation konvergieren solche Abweichungen ebenso rasant, weil wegen der exponentiellen Entwicklung des Skalenfaktors a das Produkt ρa² entsprechend zunimmt. Sollte um bei Eugen zu bleiben unser Universum räumlich sphärisch sein, dann sollte die Abweichung der relativen Dichte Ω von 1 bei ~ 10^-30 liegen und die Größe des Universums wäre damit abschätzbar. Demgegenüber schließt die heutige Datenlage eine solche Abweichung von 1% nicht aus.

Wie ist es mit der von Dir erwähnten beschleunigten Expansion? Ich nehme an, daß dann (1-Ω)/Ω) abnimmt, so klein 1 -Ω heute auch sein mag, bin aber nicht sicher.

Noch in dem Zusammenhang @Eugen
Ich kann Deine Abneigung gegen ein unendlich ausgedehntes Universum nicht nachvollziehen. Zwar führen Unendlichkeiten zu manchen physikalisch unsinnigen Aussagen. Wenn aber ein physikalisch erlaubter Prozess (andernfalls gäbe es uns nicht) Raum generiert, dann ist es doch eigentlich nicht weit hergeholt, daß so ein Prozess, der nicht am Ort x, sondern überall beginnt, nicht räumlich begrenzt ist. Es würde mich interessieren, wie Du das siehst.

Gruß, Timm

Eyk van Bommel
16.10.13, 15:03
Wenn ein kleiner Ausschnitt des Universums, in leeren Raum verbracht, diese oder jene Dynamik hätte, dann ist das auch die Dynamik des ganzen Universums.
Das ist natürlich eine Erklärung mit der man leben könnte. Wenn man sich nicht darüber weiter auseinander setzen möchte.

Wie schon mehrfach erwähnt: Die ART ist ganz einfach.
Einfach ist nun eben relativ und hängt dann noch von den Randbedingungen/Annahmen ab.

Die Gesetzte für ein Gummiband sind auch einfach. Und wenn ein Teil (lokal) sich dehnt oder zusammenzieht, dann gilt das (lokal) für das ganze (global) Gummiband.

Das Verhalten ändert sich aber „schlagartig“ wenn ich die Enden verklebe. Zumindest, dann wenn ich die Ursache für das Ziehen und Stauchen an die ART anpasse. Lokal bemerkt man keine Unterschied - Global wird sich der Radius nicht ändern.

Bei einer Hypersphäre ist der Rand der Kugel verklebt.

Soweit meine Gedanken dazu.

Vielleicht anders herum gefragt:

A) Kannst du mein "Problem" nachvollziehen? Die "Tatsache dass ich die ART wohl nicht verstanden habe" herausgenommen.

B) Wäre für dich ein mathematisch vorhergesagter „Big Crunch“ (z.B. theoretischer Wert der Vakuumenergie ist richtig) aufgrund einer geometrischen Struktur des Universums zu verhindern?

Gruß
EvB

Man kann die Dynamik eines Luftballons nicht ohne 3. Dimension verstehen.
Außer man schließt zunächst auf eine „dunkle Macht/Kraft“? :rolleyes:

Ich
17.10.13, 12:05
Kannst du mein "Problem" nachvollziehen? Die "Tatsache dass ich die ART wohl nicht verstanden habe" herausgenommen.
Ja. Du denkst dir einen lustig geformten, starren Raum, und darin denkst du dir Newtonsche Gravitationskraft, mit Fernwirkung und allem, die sich gegenseitig annulliert. Das ist dein Problem, aber wenn man dir das sagt, dann sagst du, du seist keineswegs in Newtonscher Denkweise verhaftet sondern schwebtest vielmehr in quantentoroidalen Sphären. Was in sich schon ein weiteres Problem ist.
Wäre für dich ein mathematisch vorhergesagter „Big Crunch“ (z.B. theoretischer Wert der Vakuumenergie ist richtig) aufgrund einer geometrischen Struktur des Universums zu verhindern?
Nein, nicht wenn das Universum homogen sein soll. Die Topologie hat keinen Einfluss auf die Dynamik. Habe ich aber schon zigmal gesagt.

Ein letzter Versuch, in nochmal anderen Worten: Anders als Newton kennt die ART keine Fernwirkung. Insbesondere kann die Expansion oder Kontraktion einer "Wolke" nur von der Masse in der Wolke selbst beeinfluss werden. Äußere Einflüsse können sie bestenfalls verzerren, nicht aber über Expansion oder Kontraktion bestimmen.
In einem homogenen, isotropen Universum können äußere Einflüsse noch nicht einmal das, da ist die Wolke gänzlich sich selbst überlassen.
Was an irgendeinem Ort eines solchen Universums passiert hat also überhaupt nichts damit zu tun, ob gedanklich irgendwo irgendwelche Enden verklebt werden. Es hat ausschließlich mit den Einflüssen zu tun, die auch an diesem Ort entstehen.

Man kann die Dynamik eines Luftballons nicht ohne 3. Dimension verstehen.
Außer man schließt zunächst auf eine „dunkle Macht/Kraft“?
Dann hätte man sie offensichtlich nicht verstanden.

Eyk van Bommel
18.10.13, 07:48
Ja. Du denkst dir einen lustig geformten, starren Raum,
Du hast eine starre Vorstellung meiner Vorstellung.:(

Mein Raum ist so Starr wie die Raumzeit. Oder besser - physikalisch gesehen - gibt es keinen Raum sondern nur Raumzeit(materie).

Ein Raum indem die Raumzeit(materie) nicht existiert spielt keine physikalische Rolle. Die Anzahl an theoretisch vorhandenen Räumen ist jedoch (imho) unendlich. Form und Geometrie darin wird jedoch allein durch die Raumzeit(materie)bestimmt. (Das ist Newton ?)

So wie (und hier war es nur ein Beispiel und kein quantentoroidalen Sphäre) der klassische Raum keinen Einfluss auf die Form des Orbitals im Atom hat.
Auch bei der Annahme von 10 Raumdimensionen würde sich die Form eines Orbitals nicht ändern. Ein Teilchen erhält keinen weiteren Freiheitsgrad nur weil ich eine zusätzliche Dimension annehme.

Mir geht es einzig und allein um die Fragestellung, ob in einer 3D-Raumzeit die Bewegung in einer „postulierten 4 Dimension“ (Expansion), durch eine allein 3-dimensional wirkende Gravitation verändert werden kann.

Wenn man die Expansion des Universums als eine Bewegung der 3. dimensionalen Raumzeit in einer 4. Dimension verstehen darf, dann mein(t)e ich, hat die „3D-Gravitation“ keinen Einfluss auf diese Bewegungsrichtung.

Die Expansionsgeschwindigkeit würde nicht von der Gesamtenergie abhängen.

und darin denkst du dir Newtonsche Gravitationskraft, mit Fernwirkung
Wer begründet den Friedmann mit einer newtonschen Denkweise:confused:

Fernwirkung? Meinst du damit Gravitationsfeld? Ich gehe von einer „Raumzeitkrümmung“ bzw. der resultierenden Gezeitenkraft als Ursache der Gravitation aus?

Aber das scheint keine Rolle zu spielen? Nur weil ich es sage.

Irgendwo muss ich wohl eine newtonsche Begründung eingebaut haben, die ich selbst nicht erkennen kann – wobei ich (also nicht du) die Friedmann-Gleichung bei dieser Frage, in Frage gestellt habe, da sie mir zu „newtonisch“ war.
Die Topologie hat keinen Einfluss auf die Dynamik. Habe ich aber schon zigmal gesagt.
Sagen bedeutet nicht, dass der Andere es auch versteht?
Topologie hat keinen Einfluss auf die Dynamik… Topologie hat keinen Einfluss auf die Dynamik… Topologie hat keinen Einfluss auf die Dynamik… Topologie hat keinen Einfluss auf die Dynamik..
Da schnakelt bei mir nichts. :(
Zwei Objekte auf einem „Stück Papier“ = Felsbrocken im 2D Universum, im Abstand X. Und ich schneide ein Loch in das Papier ("2D Donut") – dann hat das (k)einen wesentlichen Einfluss auf die Dynamik:confused:

Dann hätte man sie offensichtlich nicht verstanden.
So geht es mir mit der dunklen Energie.

Gruß
EvB

JoAx
18.10.13, 08:58
Alex, du musst echt mal die Phantasie abschalten, und Vorstellungskraft einschalten, um die Mathe zu verstehen.

Ein Blatt Papier mit einem Loch darin ist kein 2-Donut. Nur als Beispiel.

Das, was du da Raumzeit(materie) nennst, entspringt nur deiner Phantasie und nicht der Vorstellungskraft zur konkreten Mathematik.

Als ich geschrieben habe, dass man sich Anfangs das Universum auch ruhig eingebettet vorstellen darf, meinte ich das ungefähr so:

Wie lernt man Zahlen und einfachste mathematische Operationen? - Indem man reale Gegenstände abzählt, die man bildlich vorstellen kann. Eine imaginäre Zahl lässt sich nicht mehr bildlich vorstellen. Man muss da von den realen Objekten abstrahieren. Trotzdem kann man mit ihr arbeiten und sie ist auch sehr nützlich. Und so ist es auch mit (innerer) Krümmung. Ein (oder auch öfters) Mal eingebettet vorgestellt -> erkannt, verstanden, wie man die innere Krümmung allein in der Fläche liegend erkennt, dass es geht -> den Schritt wagen, die Einbettung als nicht erforderlich anzusehen -> von der Einbettung Abstrahieren, sie (gedanklich) "weg schmeißen".

Bauhof
18.10.13, 09:46
Mir geht es einzig und allein um die Fragestellung, ob in einer 3-D-Raumzeit die Bewegung in einer „postulierten 4 Dimension“ (Expansion), durch eine allein 3-dimensional wirkende Gravitation verändert werden kann.

Hallo Eyk,

ich favorisiere dieses Modell, dass senkrecht zum 3-D-Raum eine Art vom "Bewegung" in eine imaginäre vierte Raum-Dimension stattfindet. Und zwar deshalb favorisiere dieses Modell, weil man aus dieser Annahme die Lorentz-Transformationen herleiten kann. Aber mir ist klar, dass diese "Bewegung" in eine imaginäre vierte Raum-Dimension nicht beobachtbar ist.

Weil keine vierte Raum-Dimension wahrnehmbar ist, haben Gauß und Riemann eine Beschreibung der Raumzeit-Krümmung ohne Einbettung in eine höhere Dimension gefunden und damit die geometrische Grundlage für die ART geliefert.

Wenn man die Expansion des Universums als eine Bewegung der 3. dimensionalen Raumzeit in einer 4. Dimension verstehen darf, dann mein(t)e ich, hat die „3D-Gravitation“ keinen Einfluss auf diese Bewegungsrichtung.

Ich vermute, du meinst mit "3-D-Gravitation" die Newtonsche Gravitationstheorie. Wenn das so ist, dann hätte m.E. diese "3-D-Gravitation" keinen Einfluss auf die Bewegung der dreidimensionalen Raumzeit in einer 4. Dimension.

M.f.G. Eugen Bauhof

P.S.
Deine Beiträge sind schwer verständlich, weil man nicht weiß, auf was du hinauswillst. Ich hoffe, ich hatte diesmal mehr Glück, deinen Beitrag verstanden zu haben.

JoAx
18.10.13, 11:12
Weil keine vierte Raum-Dimension wahrnehmbar ist, haben Gauß und Riemann eine Beschreibung der Raumzeit-Krümmung ohne Einbettung in eine höhere Dimension gefunden und damit die geometrische Grundlage für die ART geliefert.


Nein, Eugen.
Nicht, weil sie nicht wahrnehmbar ist, sondern, weil sie nicht nötig ist, letztlich überflüssig, um innere Krümmung zu definieren und auch zu messen.

Eyk van Bommel
18.10.13, 11:31
Hallo Bauhof,
Deine Beiträge sind schwer verständlich,
Das gilt nicht nur hier - meine Vorträge leiden an einer ähnlich Problematik, obwohl ich mich dann "sehr gut" mit der Materie auskenne. :o
weil man nicht weiß, auf was du hinauswillst.
In algemeinen auf eine angenehme Diskussion :) Um die Uhrzeit mit einer Tasse Kaffee.
Ich hoffe, ich hatte diesmal mehr Glück, deinen Beitrag verstanden zu haben.
Ich denke schon.
Aber mir ist klar, dass diese "Bewegung" in eine imaginäre vierte Raum-Dimension nicht beobachtbar ist.
Ist mir nicht klar. Bei den Gravitonen gab es mal die Theorie, dass diese in der 4. Dimension "kurzeitig verschwinden" um die geringe Wirkung zu erklären.

Es hängt vom Modell ab, ob Messergebnisse als "Ursache einer Bewegung in einer 4. Dimension" gesehen werden können oder nicht. Du misst auch nicht die „Raumzeit(-krümmung)“ sondern die Bewegung der Teilchen.

Wenn man (nur) mit der Hinzunahmen einer 4. Dimension die Dunkle Energie (DE) erklären kann (die Bewegung der Teilchen/Galaxien), dann würde ich sagen: Die Messung der DE bzw. der Galaxien ist eine Messung der Expansion in einer 4. Raumdimension.

Ich vermute, du meinst mit "3-D-Gravitation" die Newtonsche Gravitationstheorie.
Wieso immer "Newtonsche Gravitationstheorie."? Wieso ist diese Sichtweise bei der ART nicht möglich? In wie viele Richtungen lässt sich den die Raumzeit - bezogen auf den Raum - krümmen? in 3 oder in 4?

Ich nehme die "Newtonsche Raumkugel" die ICH oben als Beispiel genommen hat – mache ein Loch rein und Blase die Kugel zu einem Ballon auf. Bis die "Newtonsche Raumkugel" nur noch als Ballonfläche wahrnehmbar ist. Das ist doch nicht "Newton"?

Spekulation an:
Z.B. Wenn die theoretische Berechnung der Vakuumenergie richtig wäre, dann müsste das Universum schon wieder kollabiert sein (wenn ich das richtig sehe).
Für mich bildet sie aber eher einen "perfekten homogenen Hintergrund/“ um die Ballonhaut schön flach zu halten.

Gruß
EvB

Eyk van Bommel
18.10.13, 11:41
Hi JoAx,
Nicht, weil sie nicht wahrnehmbar ist, sondern, weil sie nicht nötig ist, letztlich überflüssig, um innere Krümmung zu definieren und auch zu messen.
Gut das geht beim Ballon (ohne Dynamik) auch – deswegen zu behaupten die 3. Dimension gibt es nicht fände ich übertrieben :confused: :D
Nicht, weil sie nicht wahrnehmbar ist, sondern, weil sie nicht nötig ist, letztlich überflüssig, um innere Krümmung zu definieren und auch zu messen.
Wenn die Dynamik und die Geometrie vollständig durch die bekannte Raumzeit(-materie) erklärt werden könnte, dann benötigt man die 4. Raumdimension wohl nicht.
Wobei es doch genügend mehrdimensionale physikalische Ansätze gibt um ganz andere Phänomene zu beschrieben.

Wenn es eines Tages Mainstream wird, sagen wir, dass es 16 Raumdimensionen gibt um quantenphysikalische Prozesse zu erklären– ist es dann eine zusätzliche Annahme, dass sich das Universum in einer davon ausbreitet?:rolleyes:

Gruß
EvB

JoAx
18.10.13, 11:43
Ich nehme die "Newtonsche Raumkugel" die ICH oben als Beispiel genommen hat – mache ein Loch rein und Blase die Kugel zu einem Ballon auf. Bis die "Newtonsche Raumkugel" nur noch als Ballonfläche wahrnehmbar ist. Das ist doch nicht "Newton"?


Das ist weder Newton, noch ART, noch das, was "Ich" gemeint hat. (IMHO, natürlich.)

Eyk van Bommel
18.10.13, 12:10
Das ist weder Newton, noch ART, noch das, was "Ich" gemeint hat. (IMHO, natürlich.)
So war das nicht gemeint. Das ist mir klar.

Aber Kugel aus dem Universum war von ICH und Newton auch.

Die Bezeichung "Newtonsche Raumkugel" ist natürlich eher "witzig" gemeint. Und der Rest mit dem Aufblasen beruht natürlich auf meinen misst. Ich versuch(t)e einfach bildhaft zu sein.

Gruß
EvB

Bauhof
18.10.13, 13:32
Zitat von Bauhof
Weil keine vierte Raum-Dimension wahrnehmbar ist, haben Gauß und Riemann eine Beschreibung der Raumzeit-Krümmung ohne Einbettung in eine höhere Dimension gefunden und damit die geometrische Grundlage für die ART geliefert.

Nein, Eugen.
Nicht, weil sie nicht wahrnehmbar ist, sondern, weil sie nicht nötig ist, letztlich überflüssig, um innere Krümmung zu definieren und auch zu messen.

Hallo Johann,

du hast natürlich recht.
Aber ich glaube nicht, dass Eyk solche Feinheiten weiterhelfen.

M.f.G. Eugen Bauhof

JoAx
18.10.13, 13:42
Aber ich glaube nicht, dass Eyk solche Feinheiten weiterhelfen.

Die muss er aber begreifen. Und das kann er. Er darf nur nicht erwarten, dass man sich wirklich alles bildhaft vorstellen kann/muss.

Grüße

Bauhof
18.10.13, 13:54
Das gilt nicht nur hier - meine Vorträge leiden an einer ähnlich Problematik, obwohl ich mich dann "sehr gut" mit der Materie auskenne. :o

Hallo Eyk,

Oh Gott! Wer sind denn deine Zuhörer?

Bei den Gravitonen gab es mal die Theorie, dass diese in der 4. Dimension "kurzeitig verschwinden" um die geringe Wirkung zu erklären.

Sicherlich, aber das war nur eine Theorie und keine Beobachtung.

Es hängt vom Modell ab, ob Messergebnisse als "Ursache einer Bewegung in einer 4. Dimension" gesehen werden können oder nicht. Du misst auch nicht die „Raumzeit(-krümmung)“ sondern die Bewegung der Teilchen.

Nein.
Beim "Sonnenfinsternis-Experiment" wurde die Raumzeitkrümmung der Sonne gemessen in Form der Ablenkung des Lichts von Sternen.

Wieso immer "Newtonsche Gravitationstheorie."? Wieso ist diese Sichtweise bei der ART nicht möglich? In wie viele Richtungen lässt sich den die Raumzeit - bezogen auf den Raum - krümmen? in 3 oder in 4?

1. Was heißt Raumzeit - bezogen auf den Raum?
2. Was meinst du mit Krümmungs-Richtung? Es werden verschiedene Krümmungen unterschieden (z.B. positiv und negativ).

M.f.G. Eugen Bauhof

Ich
18.10.13, 14:51
Du denkst dir einen lustig geformten, starren Raum, und darin denkst du dir Newtonsche Gravitationskraft, mit Fernwirkung und allem, die sich gegenseitig annulliert.
Du hast eine starre Vorstellung meiner Vorstellung.:(
Sorry, aber das hier stand im OP:
Wäre das Universum von „Flatland“ zu einer 3D Sphäre/ oder xy- Donut gekrümmt, dann würde die Gravitation max. dazu führen können, dass sich alle Masse an einem einzelnen Punkt auf der 3D Sphäre sammelt. Das wäre dann eher so wie hier. Der „Big Crunch“ würde irgendwo beginnen und das Universum würde „in sich zusammenstürzen“, wobei Ur- und „End“-knall nicht am selben Ort stattfinden.
[...]
Ob es aber zum „Big Crunch“ bzw. „Endknall“ kommt hängt dann NICHT von der Energie/Masse im Universum ab – sondern von der Symmetrie der (Massen-)Punkte auf dem Ballon. In einem (am Ende) symmetrischen Universum würde sich die Gravitationskraft aufheben und es kommt nicht zum „Big Crunch“ bzw. „Endknall“.


Ich versuche wirklich, zu verstehen, was du so meinst. Und das war eine der wenigen Stellen, wo das einigermaßen möglich war, dachte ich. Es steht sogar explizit "Gravitationskraft" drin und "aufheben".

Vor weiterer Exegese kapituliere ich. Ich kenne manche der Wörter, die du verwendest, aber sie ergeben in ihrer Zusammenstellung keinen Sinn für mich.
Nur ein Punkt ist mir noch aufgefallen, wo ich wenigstens das Missverständnis zu erkennen glaube: diese Newtonsche Kugel von mir ist ein Ausschnitt aus unserem echten 3D Raum. Die hast du nicht aufzublasen. Sie entspricht vielmehr einer kleinen Kreisscheibe auf deiner aufgeblasenen 2D-Ballonoberfläche (die ja ein Modell für den 3D-Raum ist). Und sie macht, was sie (nach Newton) muss, ohne sich dabei um die Form des Ballons zu kümmern.

Eyk van Bommel
18.10.13, 15:13
Oh Gott! Wer sind denn deine Zuhörer?
Die üblichen Zuhörer bei wissenschaftlichen Vorträgen bei Tagungen und Kongressen - oder Laborbesprechungen... Aber ich hoffe ganz so schlimm ist es da nicht.:)
Beim "Sonnenfinsternis-Experiment" wurde die Raumzeitkrümmung der Sonne gemessen in Form der Ablenkung des Lichts von Sternen.
So kann man es sehen

Ich sehe es so, dass man eine Ablenkung des Lichts gemessen hat -Punkt.

Und es gibt ein Modell das dieses als Folge der Raumzeitkrümmung vorhergesagt hat. Ohne Einstein hätte man dies auch gemessen und man wäre nicht (automatisch) auf Raumzeitkrümmung gekommen.

Das Modell bestimmt die Interpretation der Messergebnisse. Gemessen wird die Bewegung von "Teilchen"
Was heißt Raumzeit - bezogen auf den Raum?
Raum: x,y,z und einmal „t“
Ich beziehe mich auf x, y, z und wollte t weglassen.
Es werden verschiedene Krümmungen unterschieden.
Hat diese Krümmung eine Angabe wie x,y und z?

Gruß
EvB

Nicht, weil sie nicht wahrnehmbar ist, sondern, weil sie nicht nötig ist, letztlich überflüssig, um innere Krümmung zu definieren und auch zu messen.
Was denkt ihr nur über mich? :( Der Satz hätte von mir kommen können.
Ein Blatt Papier mit einem Loch darin ist kein 2-Donut. Nur als Beispiel.
Das ist/war mir auch klar. Genauso wie man keine Hypersphäre erhält, wenn man ein Loch in "Newtonsche Raumkugel" bohrt und aufbläst.

Und ich denke ich habe auch kein Problem damit nur mit der inneren Krümmung zu leben - aber dann halte ich das Bild (mein Bild) der Hypershäre für falsch/unnötig. Und wenn es so ist - dann ist es so. Dann muss man nicht darüber diskutieren. Aber was ist wenn nicht (siehe DE).

Und wenn du als 2D-JoAx auf dem Ballon keine 3 dimension benötigst - O.k

Mit einer "Newtonsche Raumkugel" kann ich mich aber nicht wirklich anfreunden.

Bauhof
18.10.13, 15:54
Ich sehe es so, dass man eine Ablenkung des Lichts gemessen hat -Punkt. Ohne Einstein hätte man dies auch gemessen und man wäre nicht (automatisch) auf Raumzeitkrümmung gekommen.

Hallo Eyk,

falsch!
Man hat nach Newtons Interpretation nur den halben Wert berechnet und vorhergesagt. Nur mit Hilfe Einsteins Interpretation der gekrümmten Raumzeit konnte der später gemessene Wert richtig vorhergesagt werden.

Hat diese Krümmung eine Angabe wie x,y und z?

Nein. Zum Beispiel beim Kreis kann man das Krümmungsmaß (http://de.wikipedia.org/wiki/Kr%C3%BCmmung) der Kreislinie als Reziprokwert des Radius angeben.

M.f.G. Eugen Bauhof

Eyk van Bommel
18.10.13, 16:41
Hallo Bauhof,
Man hat nach Newtons Interpretation nur den halben Wert berechnet...
Darum geht es nicht. Ich wusste nicht, dass man vorher (vor Einstein) überhaupt so etwas berechnet hat.
Ich habe geschrieben,
Ohne Einstein hätte man dies auch gemessen
Ich spreche hier nicht von Interpretation. Ich messe immer die Bewegung eines Teilchens. Das Messergebnis wird dann interpretiert. Wenn man will - mus man aber garnicht.

Auch die Erklärung nach Newton ist eine Interpretation. Der hatte aber gar kein Modell - daher hmm. Hat auch funktiniert Mathe ohne Modell.

ART ohne Modell geht imho auch.

Wenn man eines Tages eine Grenze für die Gültigkeit der ART findet, dann wird die Mathematik genuso noch gültig sein wie jetzt, genauso wie die Gesetzte von Newton noch gültig sind. Nur die Ursache für die Bewegungsänderung muss keine Raumkrümmung mehr sein. Dann aber auch dort nicht mehr wo die Gesetze der ART selbst noch gültig sind.

Kreislinie als Reziprokwert des Radius angeben.
Benötigt man für r nicht x,y,z? Für einen Vektor benötigt man doch mind. x und y? Und für das r einer Hypersphäre x,y,z und...?

Gruß
EvB

Eyk van Bommel
18.10.13, 19:25
@JoAx
Verstehe ich dich richtig, dass ich mir das Ganze als eine imaginäre Hypersphäre vorstellen soll.
@ICH
Danke für deine bisherigen Versuche. Leider werde ich immer wieder missverstanden. Und je mehr ich versuche das Problem zu schildern, desto mehr fühle ich mich von Euren versuchen mir zu helfen beinahe beschämt. Bauhof kann mir noch am besten folgen, was wie ich befürchte kein Kompliment zu sein scheint.;)
Sie entspricht vielmehr einer kleinen Kreisscheibe auf deiner aufgeblasenen 2D-Ballonoberfläche (die ja ein Modell für den 3D-Raum ist). Und sie macht, was sie (nach Newton) muss, ohne sich dabei um die Form des Ballons zu kümmern.
Dem habe ich nichts hinzu zufügen. Dem stimme ich zu.:)

Sie kümmert sich nicht um die Form des Ballons und genauso wenig kümmert es die Form des Ballons.
Und wenn du 100 solche Flächen nimmst – dann gilt das auch. Und wenn du 1000… Wenn du den Ballon lückenlos zupflasterst mit diesen Flächen, dann ändert dies nichts an der Form des Ballons. Damit ist auch der (imaginäre) Radius gemeint.

Ich denke da sind wir uns einig?

Wo wir uns nicht einig sind ist: Wenn einer dieser Flächen lang ziehst – gaanz lang, so lang, dass du sie einmal "um den Äquator" ziehst und auf der Rückseite zusammentackerst, dann kann sich die Fläche nicht mehr nach Newton verhalten.

Gruß
EvB

Eyk van Bommel
18.10.13, 19:50
Wo wir uns nicht einig sind ist: Wenn einer dieser Flächen lang ziehst – gaanz lang, so lang, dass du sie einmal "um den Äquator" ziehst und auf der Rückseite zusammentackerst, dann kann sich die Fläche nicht mehr nach Newton verhalten.
Ist diese Sichtweise falsch? Muss ich mir das so vorstellen, dass während ich die Fläche ziehe – wieder viele „kleine Flächen“ entstehen.

Ich
18.10.13, 19:55
Wo wir uns nicht einig sind ist: Wenn einer dieser Flächen lang ziehst – gaanz lang, so lang, dass du sie einmal "um den Äquator" ziehst und auf der Rückseite zusammentackerst, dann kann sich die Fläche nicht mehr nach Newton verhalten.

Du darfst den Kreis nicht langziehen, dann erfüllt er seine Funktion nicht mehr. Der Sinn dieser Flächen ist es eben, keine komische Topologie zu haben. Sie sind und waren immer kleine Ausschnitte aus dem Ganzen. Per Definitionem so klein, dass sogar noch die Newtonschen Formeln auf sie anwendbar sind.
Das ist genau das, was du immer ignorierst: Die ART funktioniert ohne Rücksicht auf das Ganze, sie funktioniert lokal. Wenn alles symmetrisch ist, dann macht dieser Kreis genau das, was von ihm erwartet wird, und es ist ihm egal, wo er eingebaut ist.

Wenn du also den Luftballon mit 1000 solchen Kreisen (oder besser Flächen) lückenlos zupflasterst, und einer dieser Kreise soll nach Newton kleiner werden, dann wird er das. Punkt. Und weil alles symmetrisch ist, werden die anderen genauso kleiner. Und weil sie den Luftballon immer noch lückenlos parkettieren, ist dieser auch kleiner geworden.
Das Ganze folgt dem kleinen, nicht andersrum.

Eyk van Bommel
18.10.13, 20:53
Du darfst den Kreis nicht langziehen, dann erfüllt er seine Funktion nicht mehr. Der Sinn dieser Flächen ist es eben, keine komische Topologie zu haben. Sie sind und waren immer kleine Ausschnitte aus dem Ganzen. Per Definitionem so klein, dass sogar noch die Newtonschen Formeln auf sie anwendbar sind.
Das ist genau das, was du immer ignorierst: Die ART funktioniert ohne Rücksicht auf das Ganze, sie funktioniert lokal. Wenn alles symmetrisch ist, dann macht dieser Kreis genau das, was von ihm erwartet wird, und es ist ihm egal, wo er eingebaut ist.
Das wollte ich mit meiner letzen Antwort sagen. Das habe ich nun verstanden.
Danke.:)
Wenn du also den Luftballon mit 1000 solchen Kreisen (oder besser Flächen) lückenlos zupflasterst, und einer dieser Kreise soll nach Newton kleiner werden, dann wird er das. Punkt. Und weil alles symmetrisch ist, werden die anderen genauso kleiner.
O.K
Und weil sie den Luftballon immer noch lückenlos parkettieren, ist dieser auch kleiner geworden.
Da bin ich draußen. Da für mich hier eine "Nettobewegung" des Ballons entsteht, die durch die Flächenbewegung nicht abgedeckt ist.

Kam in Mathe nicht bis zur ART nur bis zum Vektor rechnen und ich bekomme den resultierenden Pfeil für die Nettobewegung nicht hin.

Brauche nun eine Weile.

Aber ich habe es gecheckt - hoffe ich :)

Ich
18.10.13, 21:26
Kam in Mathe nicht bis zur ART nur bis zum Vektor rechnen und ich bekomme den resultierenden Pfeil für die Nettobewegung nicht hin.
Es gibt keinen solchen Pfeil.
Genau deswegen hatte ich vorher mal auf den anderen Thread verlinkt, dort war das Ganze dargestellt.
Wenn du dir irgendeinen beliebigen Punkt auf der Oberfläche herausgreifst, dann werden alle Punkt in der Umgebung sich auf diesen zu bewegen. Wenn du stattdessen einen benachbarten Punkt herausgreifst, dann bewegt sich die Nachb*****aft auf diesen zu.
Bei der Bewältigung dieses Rätsels helfen uns zwei Grundprinzipien der Relativitätstheorien:
1. Geschwindigkeit ist relativ - je nachdem, von wo aus ich schaue, haben die Punkte unterschiedliche Geschwindigkeiten. Man kann ihnen keinen eindeutigen Geschwindigkeitsvektor zuordnen.
2. Gravitationsbeschleunigung ist relativ - je nachdem, von wo aus ich schaue, haben die Punkte unterschiedliche Beschleunigungen. Man kann ihnen auch keinen eindeutigen Beschleunigungsvektor zuordnen.

Was man stattdessen macht ist, einen Schritt über Vektoren hinauszugehen zu den Tensoren (http://de.wikipedia.org/wiki/Tensor).
Vektoren sind Tensoren erster Stufe, und sie helfen hier offensichtlich nur bedingt weiter. Tensoren zweiter Stufe sind das, was wir brauchen: Sie beschreiben nicht, in welche Richtung sich Dinge bewegen oder beschleunigen. Sie beschreiben, ob sich Dinge aufeinander zu oder voneinander weg bewegen oder beschleunigen. In unserem Fall ist es nicht erforderlich oder möglich, absolute Geschwindigkeits- oder Beschleunigungsvektoren zu vergeben. Aber wir können mit Sicherheit sagen, dass die Dinge aufeinander zu beschleunigen. Und das überall auf dem gesamten Ballon in demselben Ausmaß, also homogen.

Der Name Tensor kommt von "Spannung", und das passt hier auch ausgesprochen anschaulich herein: denke dir den Luftballon, aus dem gerade Luft ausgelassen wird. Seine Haut steht unter Spannung. Deswegen wird sich die Haut (in der Fläche selbst betrachtet) zusammenziehen: Benachbarte Punkte beschleunigen aufeinander zu. Man muss weder sagen, welcher Punkt sich im absoluten Sinn wohin bewegt, noch wohin er beschleunigt, aber man kann mit mathematischer Präzision und Eindeutigkeit angeben, dass die Beschleunigung "aufeinander zu" ist.
Und genau so funktioniert die ART. Geschwindigkeiten und Beschleunigungen als Vektoren benötigen eine Richtung und sind abhängig vom Bezugssystem. Expansion und Kontraktion hingegen beschränken sich auf "aufeinander zu" bzw. "voneinander weg" und sind deswegen für alle Fälle anwendbar. Auch wenn man sich, wie bei der Ballonoberfläche, keine Richtungen mehr vorstellen kann oder will. Die sind nicht nötig.

Eyk van Bommel
19.10.13, 14:37
Hallo ICH,
Vorweg ich denke ich habe verstanden. Mein Eingangsgedanke ist falsch gewesen und ich werde es nicht mehr behaupten :)
Sollte es so rüber kommen - jetzt oder in Zukunft, dann habe ich was nicht richtig umgesetzt.
Es gibt keinen solchen Pfeil.
Ich glaube du meinst einen anderen. In den 3 Dimensionen gibt es keinen Pfeil. Wenn der Ballon kleiner wird – dann ist auch für mich klar, dass alle dasselbe sehen. Danke aber für die Tensoren-Einführung.

Ich meinte die Nettobewegung „in der 4. Dimension“ – der Radius wird kleiner. Und der Radius steht senkrecht auf der Raumzeit. Die Massen stürzen nicht nur auf einander zu, sondern auch noch „nach innen“. Oder ist dies keine Bewegung. Oder sollte ich tatsächlich "r" eher wie "i" verstehen.
----------------
Tensor – Spannung:
Ich hätte die Spannung eher gegenüber einer Krümmung gesehen – nicht auf Zug.
Auch jetzt noch. Schaut man sich die DE an so spricht man von Raum „Zuwachs“ – Der Raum dehnt sich/wächst. Ohne jegliche Form des „Widerstandes“.

Warum sollte umgekehrt anders sein?

Wenn ich das Beispiel mit den lokalen Flächen sehe und den Kosmos, dann habe ich das Gefühl, dass sich die Flächen zusammen ziehen können und dazwischen „in den Lücken“ sich Voids bilden würden.

Sei mir nicht böse – aber ich habe ja schon einiges angenommen ( von dem was du erklärt hast.)

Aber: Dass die Flächen kleiner werden und „dazwischen“ leerer Raum „entsteht“ wie wir es von der DE her kennen ist…? Immer noch völlig falsch oder innerhalb des erlaubten.

Zumindest für die Beobachter wäre auch hier das Bild für alle gleich.

---------
Vielleicht muss ich es noch ein bisschen wirken lassen.

Gruß
EvB

Ich
20.10.13, 20:19
Ich meinte die Nettobewegung „in der 4. Dimension“ – der Radius wird kleiner. Und der Radius steht senkrecht auf der Raumzeit.
Lass diesen Käse einfach, das hilft doch nichts. Beim flachen Universum wäre die Radiusänderung dann unendlich, beim hyperbolischen gleichzeitig positiv und negativ, beim Torus irgendwas. Man kann noch nicht mal alle Räume in nur eine weitere Dimension einbetten. Das Universum ist kein Luftballon. Hör' auf, es wie einen zu behandeln, das mag es nicht.
Tensor – Spannung:
Ich hätte die Spannung eher gegenüber einer Krümmung gesehen – nicht auf Zug.
Ich hab' da ausdrücklich vom Luftballon gesprochen, nicht vom Universum. In den Ingenieurswissenschaften gibt es z.B. den isotropen Spannungszustand, wo alles sich ohne bevorzugte Richtung zusammenziehen will. Das ist äußerst anschaulich für die Richtungslosigkeit der Expansion oder Kontraktion. Übrigens bettet kein Konstrukteur seine Werkstücke in vierte Dimensionen ein, um das zu verstehen. Er versucht sich stattdessen an der Realität.
(Bitte nicht beachten: tatsächlich scheint das Vakuum "unter Spannung" zu stehen. Das nennt sich "kosmologische Konstante" und führt - entgegen der Vorstellung - zu beschleunigter Expansion. Das tut aber jetzt nichts zur Sache, der Tensor sollte nur die Richtungslosigkeit rüberbringen, sonst nichts.)
Wenn ich das Beispiel mit den lokalen Flächen sehe und den Kosmos, dann habe ich das Gefühl, dass sich die Flächen zusammen ziehen können und dazwischen „in den Lücken“ sich Voids bilden würden.
Wenn wir vom homogenen Modell ausgehen, ist das nicht möglich. Bei Anwesenheit von Inhomogenitäten kommt es natürlich zu Strukturbildung, aber die ändert die Universumsdynamik nicht. Bloß weil zwei Regionen (bei gleicher Masse) stärker konzentriert sind, verschwindet doch nicht plötzlich die gegenseitige Anziehung. Die bleibt natürlich.
Deine dichten Regionen sind dann nichts weiter als kleine Massepünktchen in einem Modell, das ziemlich genau dasselbe macht wie homogen verteilter Staub. Es mag dich interessieren, dass man Universumssimulationen durchaus auch mit Galaxien als Staubkörnchen rechnet. Ab einer gewissen Größenskala verschwindet die K****igkeit einfach wieder.

Eyk van Bommel
22.10.13, 14:14
Hallo ICH,
Das Universum ist kein Luftballon. Hör' auf, es wie einen zu behandeln, das mag es nicht
Ich denke ja nicht, dass ich der einzige bin, der sich an das Bild einer Hypersphäre als mögliches Bild des Universums gewöhnt hat (oder Donut).

Dass dieses Bild nun soweit weg von der eigentlich physikalischen Aussage ist - wer hätte das gedacht.

Entschuldige daher – dass ich mich nur langsam wieder davon lösen kann. Bin aber dabei.
Beim flachen Universum wäre die Radiusänderung dann unendlich, beim hyperbolischen gleichzeitig positiv und negativ, beim Torus irgendwas.
Natürlich hat die Form Einfluss auf das Gesagte. Ich sehe das Problem natürlich eher allgemein. Radius "zieht" eben nur bei einer Hypersphäre. Die Frage war: Erzeugt die Spannung bzw. ihre Änderung eine (Netto)Bewegung in einer weiteren Dimension.
Hör' auf, es wie einen zu behandeln, das mag es nicht.
Das Universum behandelt mich auf so viele Weisen die ich nicht mag – das muss es abkönnen.:D
..Richtungslosigkeit der Expansion oder Kontraktion...
Bitte entschuldige mein Unverständnis und dem ggf. wiederkehrenden Argument.

Die Richtungslosigkeit der Expansion oder Kontraktion bezieht sich imho nur auf 3D. Bei einem Universum das (ggf.) in eine höhere Dimension eingebettet ist, sehe ich zumindest (noch) eine feste Richtung.

Aber nur natürlich nur, wenn das Universum das ist was es nicht gerne ist – ein Ballon, Sattel oder Donut :)
Übrigens bettet kein Konstrukteur seine Werkstücke in vierte Dimensionen ein, um das zu verstehen.
Nein -Er müsste es auch mit 2 Dimensionen versuchen. Und hast du nicht geschrieben, dass man die Bewegung (luft raus lassen/aufblasen) ohne 3. Dimension bei einem Ballon als „Flatlander“ nicht verstehen würde?
Tatsächlich scheint das Vakuum "unter Spannung" zu stehen. Das nennt sich "kosmologische Konstante" und führt - entgegen der Vorstellung - zu beschleunigter Expansion.
Ist bekannt – aber das ist dasselbe in grün nur wäre hier mein Problem „ das der Radius größer" wird. Es bleibt (noch) eine Nettobewegung in einer höheren Dimension. Auch wenn der Tensor keine Richtung vor gibt. So sollte er doch auch keine Richtung in einer höheren Dimension erzeugen?

Bitte verstehe meine Worte nicht als Widerworte. Sondern als Verständnisproblem.

Oder anders und bitte nicht hauen: Bauchgefühl

Würde man eine Bewegung in einer höheren Dimension bei fehlender Kenntnis dieser - nicht auf einen Tensor/innere Spannung schließen?
Tensor sollte nur die Richtungslosigkeit rüberbringen, sonst nichts
Ich hoffe ich besitze keine tensorische Aufnahmefähigkeit. :o
Wenn wir vom homogenen Modell ausgehen, ist das nicht möglich
Habe nachgedacht und denke du hast recht.

Gruß
EvB

Ich
24.10.13, 15:10
Hallo EvB,

Die Frage war: Erzeugt die Spannung bzw. ihre Änderung eine (Netto)Bewegung in einer weiteren Dimension.
Eine Änderung der Krümmung einer homogenen, positiv gekrümmten Fläche entspricht einer Änderung des Radius ihrer Einbettung in R3. Das siehst du wohl als Bewegung.
Negativ gekrümmte Flächen können nicht so betrachtet werden, weil für die kene Einbettung in R3 existiert. Diese "Bewegung in einer weiteren Dimension" ist also eine Eigenschaft von Kugeloberflächen und nicht von allgemeinen Flächen oder Räumen. Hat also vermutlich weniger mit Physik zu tun als mit Luftballons.
Das Universum behandelt mich auf so viele Weisen die ich nicht mag – das muss es abkönnen.
Da muss ich dir Recht geben.
Und hast du nicht geschrieben, dass man die Bewegung (luft raus lassen/aufblasen) ohne 3. Dimension bei einem Ballon als „Flatlander“ nicht verstehen würde?
Ich habe geschrieben, dass man die Dynamik nicht verstehen würde. Weil man den Luftdruck als Ursache für die Expansion nicht bemerken würde.
Auch wenn der Tensor keine Richtung vor gibt. So sollte er doch auch keine Richtung in einer höheren Dimension erzeugen?
Der Tensor, den ich meinte - der Expansionstensor -, ist ein dreidimensionales Objekt, weil es das Verhalten des Raumes beschreibt, nicht der Raumzeit. Um ihn zu definieren, muss man u. A. eine Zeitrichtung definieren, zu der dann der betrachtete Raum senkrecht steht. Wenn dir das irgendwie weiterhilft.

Eyk van Bommel
28.10.13, 09:44
Hallo ICH,
ich arbeite noch an dem Thema und beschäftige mich weiter mit deinen Aussagen. Und manchmal dauert alles ein bisschen länger.

Gruß
EvB